Vous êtes sur la page 1sur 104

The problems in this booklet are organized into strands.

A
problem often appears in multiple strands. The problems are
suitable for most students in Grade 7 or higher.

WWW.C E M C .U WAT E R LO O.C A | T h e C E N T R E fo r E D U C AT I O N i n M AT H E M AT I C S a n d CO M P U T I N G

Problem of the Week


Problem C
Lost Marbles Found
Mark has a bag that contains exactly 3 black marbles, 6 gold marbles, 2 purple
marbles and 6 red marbles.
Mark finds a number of white marbles and adds them to the bag. He tells Sam
that if she now draws a marble at random from the bag, the probability of it
3
being black or gold is .
7
How many white marbles did Mark add to the bag?

WWW.C E M C .U WAT E R LO O.C A | T h e C E N T R E fo r E D U C AT I O N i n M AT H E M AT I C S a n d CO M P U T I N G

Problem of the Week


Problem C and Solution
Lost Marbles Found
Problem
Mark has a bag that contains exactly 3 black marbles, 6 gold marbles, 2 purple marbles and 6
red marbles. Mark finds a number of white marbles and adds them to the bag. He tells Sam
that if she now draws a marble at random from the bag, the probability of it being black or
3
gold is . How many white marbles did Mark add to the bag?
7

Solution
In order to determine the probability that a marble drawn from the bag is black or gold, we
divide the number of black and gold marbles in the bag by the total number of marbles in the
bag. In other words,
Probability of selecting a black or gold marble =

Number of black or gold marbles


Total number of marbles

When Mark adds white marbles to the bag, this does not change the number of black or gold
marbles. Therefore, the number of black or gold marbles in the bag is 3 + 6 = 9.
3
We are also given that the probability of drawing a black or gold marble is .
7
So the equation
Probability of selecting a black or gold marble =

Number of black or gold marbles


Total number of marbles

becomes
3
9
=
7
Total number of marbles
Since

3
9
= , this tells us
7
21
9
9
=
21
Total number of marbles

Therefore, the total number of marbles is 21.


Originally, there were 3 + 6 + 2 + 6 = 17 marbles in the bag. Then Mark added some white
marbles. Since the total number of marbles in the bag after adding some white marbles is 21,
Mark must have added 21 17 = 4 white marbles.

WWW.C E M C .U WAT E R LO O.C A | T h e C E N T R E fo r E D U C AT I O N i n M AT H E M AT I C S a n d CO M P U T I N G

Problem of the Week


Problem C
Statistics for the New Year
Integers greater than 1000 are created using the digits 2, 0, 1, 4 exactly once in
each integer.
What is the difference between the average of the largest and smallest of these
integers and the median of all the integers that can be created this way?

WWW.C E M C .U WAT E R LO O.C A | T h e C E N T R E fo r E D U C AT I O N i n M AT H E M AT I C S a n d CO M P U T I N G

Problem of the Week


Problem C and Solution
Statistics for the New Year
Problem
Integers greater than 1000 are created using the digits 2, 0, 1, 4 exactly once in each integer.
What is the difference between the average of the largest and smallest of these integers and the
median of all the integers that can be created this way?

Solution
First, we will determine the average of the largest and smallest numbers.
With a given set of four digits, the largest possible integer that can be formed puts the largest
digit in the thousands place, the second largest digit in the hundreds place, the third largest
digit in the tens place, and the smallest digit in the units place. That is, the largest integer is
formed by putting the digits in decreasing order from the thousands place to the units place.
This is because the largest digit can make the largest contribution in the place with the most
value. Thus, the largest integer that can be formed with the digits 2, 0, 1, 4 is 4210.
Similarly, with a given set of digits the smallest integer is formed by putting the digits in
increasing order from the thousands place to the units place. Here, there is an added wrinkle
that the integer must be at least 1000. Therefore, the thousands digit is at least 1. The
smallest integer of this type that can be made uses a thousands digit of 1, and then lists the
remaining digits in increasing order. This integer is 1024.
Therefore, average of the largest and smallest numbers is

4210+1024
2

5234
2

= 2617.

Now, lets determine the median of all of the numbers that can be created.
Since the thousands digit is at least 1, the smallest numbers will begin with a 1. The
remaining digits will be 0, 2 and 4, in some order. There are six possibilities for the remaining
digits: 024, 042, 204, 240, 402, or 420. Therefore, there are six numbers that begin with a 1.
Similarly, there are six numbers that will begin with a 2 and six numbers that will begin with a
4. Therefore, there are 18 different numbers that can be created this way. The smallest six
numbers will begin with a 1 and the largest six numbers will begin with a 4. The middle six
numbers will begin with a 2 and the median of all of the numbers will equal the median of the
middle six numbers. The middle six numbers, from smallest to largest, are
2014, 2041, 2104, 2140, 2401, 2410
Since there is an even number of numbers, the median of these six numbers is the average of
= 4244
= 2122. Therefore, the median of all of the numbers is
the middle two, which is 2104+2140
2
2
2122. (Alternatively, we could have listed out all 18 numbers that can be created this way in
order to determine the median.)
The difference between the average of the largest and smallest numbers and the median of all
numbers is therefore 2617 2122 = 495.

WWW.C E M C .U WAT E R LO O.C A | T h e C E N T R E fo r E D U C AT I O N i n M AT H E M AT I C S a n d CO M P U T I N G

Problem of the Week


Problem C
Can You Compete?
Gregory, Adam and Paul are athletes who competed in the downhill skiing
event in the Winter Olympics.
Gregory, Adam and Paul each finished in first, second or third. There were no
ties.
Each athlete is also from a different country. One is from Canada, one is from
France and one is from Japan.
Using the following clues, determine who placed first, second and third, and for
which country each athlete was competing.
1. Gregory was faster than Adam.
2. Gregory is not Canadian, and he did not finish in second place.
3. The Japanese athlete was faster than the French athlete.
4. Adam is not Japanese and he did not finish in third place.
5. The Canadian athlete was faster than the French athlete.
You may find the following table a helpful way to organize your solution to this
problem.
Canada France Japan
Gregory
Adam
Paul
First
Second
Third

First

Second

Third

WWW.C E M C .U WAT E R LO O.C A | T h e C E N T R E fo r E D U C AT I O N i n M AT H E M AT I C S a n d CO M P U T I N G

Problem of the Week


Problem C and Solution
Can You Compete?
Problem
Gregory, Adam and Paul are athletes who competed in the downhill skiing event in the Winter
Olympics. Gregory, Adam and Paul each finished in first, second or third. There were no ties.
Each athlete is also from a different country. One is from Canada, one is from France and one
is from Japan. Using the following clues, determine who placed first, second and third, and for
which country each athlete was competing.
1. Gregory was faster than Adam.
2. Gregory is not Canadian, and he did not finish in second place.
3. The Japanese athlete was faster than the French athlete.
4. Adam is not Japanese and he did not finish in third place.
5. The Canadian athlete was faster than the French athlete.

Solution
In our solution, we will we go through each clue and update the table based on the information
in the clue. We will put an X in a cell if the combination indicated by the row and column for
that cell is not possible, or a X if it must be true.
From clue (1), since Gregory was faster than Adam, we know that Adam could not have
finished first and that Gregory did not finish third. We can therefore put an X in the cells
corresponding to Adam in first and Gregory in third.
From clue (2), we can put an X in the cells corresponding to Gregory being Canadian and also
to Gregory in second. The table is updated below.

Gregory
Adam
Paul
First
Second
Third

Canada
X

France

Japan

First

Second
X

Third
X

We see that Gregory must have finished first. We can add a X to the corresponding cell in the
table. Since Gregory finished first and there were no ties, we know that Paul did not finish
first. We can add an X to the corresponding cell in the table. The table is updated and shown
on the top of the next page.

WWW.C E M C .U WAT E R LO O.C A | T h e C E N T R E fo r E D U C AT I O N i n M AT H E M AT I C S a n d CO M P U T I N G

Gregory
Adam
Paul
First
Second
Third

Canada
X

France

Japan

First
X
X
X

Second
X

Third
X

From clue (3), we know the French athlete did not finish first and that the Japanese athlete
did not finish third. We can therefore put an X in the cells corresponding to the French athlete
in first and the Japanese athlete in third. Since we now know that Gregory came in first, this
clue also tells us that Gregory is not French. We can put an X in the corresponding cell. The
table is updated below.

Gregory
Adam
Paul
First
Second
Third

Canada
X

France
X

Japan

First
X
X
X

Second
X

Third
X

X
X

We see that Gregory must compete for Japan. Since Gregory finished first, this also tells us
that the Japanese athlete finished first. We can add a X to the corresponding cells in the table.
Gregory
Adam
Paul
First
Second
Third

Canada
X

France
X

Japan
X

First
X
X
X

Second
X

Third
X

Since the athletes are from different countries, we now know that Adam and Paul are not from
Japan, and so we can add Xs to the corresponding cells in the table. Since we know the
Japanese athlete finished first, he could not have finished second too, and the Canadian did
not finish first, so we can add Xs to the corresponding cells in the table. The table is updated
below.
Gregory
Adam
Paul
First
Second
Third

Canada
X

France
X

Japan
X
X
X
X
X
X
2

First
X
X
X

Second
X

Third
X

WWW.C E M C .U WAT E R LO O.C A | T h e C E N T R E fo r E D U C AT I O N i n M AT H E M AT I C S a n d CO M P U T I N G

From clue (4), we can add an X in the cells corresponding to Adam finishing in third. The
table is updated as follows.

Gregory
Adam
Paul
First
Second
Third

Canada
X

France
X

Japan
X
X
X
X
X
X

First
X
X
X

Second
X

Third
X
X

We can now see that Adam must have finished in second and Paul must have finished in third.
We can add the corresponding Xs and Xs. The table is updated below.
Gregory
Adam
Paul
First
Second
Third

Canada
X

France
X

Japan
X
X
X
X
X
X

First
X
X
X

Second
X
X
X

Third
X
X
X

From clue (5), we know the Canadian athlete was faster than the French athlete. Since the
Japanese athlete finished first, this means the Canadian must have finished second and the
French athlete must have finished third. Therefore, Adam is the Canadian and Paul is the
French athlete. We can add the corresponding Xs and Xs. The table is updated as follows.
Gregory
Adam
Paul
First
Second
Third

Canada
X
X
X
X
X
X

France
X
X
X
X
X
X

Japan
X
X
X
X
X
X

First
X
X
X

In conclusion:
Gregory competed for Japan and finished in first place.
Adam competed for Canada and finished in second place.
Paul competed for France and finished in third place.

Second
X
X
X

Third
X
X
X

WWW.C E M C .U WAT E R LO O.C A | T h e C E N T R E fo r E D U C AT I O N i n M AT H E M AT I C S a n d CO M P U T I N G

Problem of the Week


Problem C
Just Perfect!
You decide to play the Red - Blue Card Game at the local fair. There are five
red cards, each with one of the numbers 1, 3, 5, 9 or 12 printed on it. Each
number is used exactly once. There are four blue cards, each with one of the
numbers 1, 3, 4 or 5 printed on it. Each number is used exactly once. You draw
one red card and one blue card. If the product of your two numbers is a perfect
square, you have drawn a winning pair. What is the probability of drawing a
winning pair?
A perfect square is an integer that can be created by multiplying an integer by
itself. For example, the number 25 is a perfect square since it can be expressed
by 5 5 or 52 .

WWW.C E M C .U WAT E R LO O.C A | T h e C E N T R E fo r E D U C AT I O N i n M AT H E M AT I C S a n d CO M P U T I N G

Problem of the Week


Problem C and Solution
Just Perfect!
Problem
You decide to play the Red - Blue Card Game at the local fair. There are five red cards, each
with one of the numbers 1, 3, 5, 9 or 12 printed on it. Each number is used exactly once. There
are four blue cards, each with one of the numbers 1, 3, 4 or 5 printed on it. Each number is
used exactly once. You draw one red card and one blue card. If the product of your two
numbers is a perfect square, you have drawn a winning pair. What is the probability of
drawing a winning pair?

Solution
In order to determine the probability, we must determine the number of ways to obtain a
perfect square and divide it by the total number of possible selections of one red card and one
blue card. We will compile this information in a table.
hhhh
h

hhhh
Number on Red Card
hhhh
hhh
hhhh
hhh
Number on Blue Card
hh

1
3
4
5

12

1
3
4
5

3
9
12
15

5
15
20
25

9
27
36
45

12
36
48
60

The numbers in the top row of the table, namely, 1, 3, 5, 9, and 12, correspond to the possible
values on the red card. The numbers in the first column of the table, namely, 1, 3, 4, and 5,
correspond to the possible values on the blue card. The other numbers in the table correspond
to the product of the number on the blue card and the number on the red card.
From the table, we see that 20 products are formed. The number 1 is a perfect square, (1 1),
and it occurs one time. The number 4 is a perfect square, (2 2), and it occurs one time. The
number 9 is a perfect square, (3 3), and it occurs two times. The number 25 is a perfect
square, (5 5), and it occurs one time. The number 36 is a perfect square, (6 6), and it
occurs two times. Seven of the products are perfect squares. Therefore, the probability of
7
drawing a winning pair is 20
.
A game is considered fair if the probability of winning is 50%. In this game, the winning
probability, as a percentage, is 35%. Can you modify the game so that it is fair?

WWW.C E M C .U WAT E R LO O.C A | T h e C E N T R E fo r E D U C AT I O N i n M AT H E M AT I C S a n d CO M P U T I N G

Problem of the Week


Problem C
Going My Way?
In a computer game, Super Maria travels through the maze shown from Start,
S, to Finish, F . She is allowed to travel only east, south, or southeast along a
path.
How many different routes can Maria take from Start to Finish?

S
N

S
F

WWW.C E M C .U WAT E R LO O.C A | T h e C E N T R E fo r E D U C AT I O N i n M AT H E M AT I C S a n d CO M P U T I N G

Problem of the Week


Problem C and Solution
Going My Way?
Problem
In a computer game, Super Maria travels
through the maze shown from Start, S, to
Finish, F . She is allowed to travel only east,
south, or southeast along a path. How many
different routes can Maria take from Start to
Finish?

S
N

S
F

Solution
We could solve this problem by tracing out
different routes and counting how many we
find. We will set up a systematic approach to
doing so, to ensure that we do not miss any
routes.
We begin by labelling the seven unlabelled
intersections in the maze as A, B, C, D, E, G,
and H, as shown.

Since Maria can only travel east, south, or southeast along a path, starting at
S she has only two choices as to where to go next: A or C.

WWW.C E M C .U WAT E R LO O.C A | T h e C E N T R E fo r E D U C AT I O N i n M AT H E M AT I C S a n d CO M P U T I N G

Case 1: Maria travels from S to A

Again, since Maria can only travel east, south, or


southeast along a path, she has only two choices as to
where to go next: B or D.
If Maria travels to B, then since she can only travel east,
south or southeast, she must go to E next, followed by
F . Therefore, one route from S to F is SABEF .
If Maria travels to D, then since she can travel east,
south or southeast, she can go to E, F or H next.

If she travels from D to E, she must then go to F . Therefore, one route from S to F is
SADEF .
If she travels from D to F , we have found another route. Therefore, one route from S to
F is SADF .
If she travels from D to H, she must then go to F . Therefore, one route from S to F is
SADHF .
Therefore, there are four routes from S to F in which Maria first travels from S to A. They
are SABEF , SADEF , SADF , and SADHF .
Case 2: Maria travels from S to C
Since Maria can travel east, south or southeast, she could travel to D, H or G next.
If she travels from C to D, she again has 3 choices: travel from D to E, F or H. We get
three different routes: SCDEF , SCDF and SCDHF .
If she travels from C to H, from H she must go to F . We have found another route,
SCHF .
If she travels from C to G, she must then go to H then F . We have found another route,
SCGHF .
Therefore, there are five routes from S to F in which Maria first travels from S to C. They are
SCDEF , SCDF , SCDHF , SCHF , and SCGHF .
In total, there are 4 + 5 = 9 different routes that Maria can take from S to F .
For further thought:
How would your answer change if Maria were allowed to also go southwest? (She is never
allowed to travel north, northeast, northwest, or west.)

WWW.C E M C .U WAT E R LO O.C A | T h e C E N T R E fo r E D U C AT I O N i n M AT H E M AT I C S a n d CO M P U T I N G

Problem of the Week


Problem C
Tipped Triangles
Two equilateral triangles, 4ABC and 4P QR, have their bases AB and P Q
sitting on line segment M N , as shown.
R
C

4ABC is tipped clockwise 65 about point B so that M BA = 65 and point


B remains where it is on M N . 4P QR is tipped counterclockwise 75 about
point P so that N P Q = 75 and point P remains where it is on M N .
Q
A

C
Y

65

75

As a result of tipping the two triangles, 4ABC intersects 4P QR such that


AC and BC intersect RP at X and Y , respectively. Vertex C of 4ABC lies
inside 4P QR.
Determine the measure of CXY .

WWW.C E M C .U WAT E R LO O.C A | T h e C E N T R E fo r E D U C AT I O N i n M AT H E M AT I C S a n d CO M P U T I N G

Problem of the Week


Problem C and Solution
Tilted Triangles
Problem

R
C

Two equilateral triangles, 4ABC and 4P QR, have


their bases AB and P Q sitting on line segment
M N , as shown.
4ABC is tipped clockwise 65 about point B so
that M BA = 65 and point B remains where it
is on M N . 4P QR is tipped counterclockwise 75
about point P so that N P Q = 75 and point P
remains where it is on M N . As a result of tipping
the two, 4ABC intersects 4P QR such that AC
and BC intersect RP at X and Y , respectively.
Vertex C of 4ABC lies inside 4P QR. Determine
the measure of CXY .

Q
Q

C
Y

65

75

Solution
In any equilateral triangle, all sides are equal in length and each angle measures 60 . Since
4ABC and 4P QR are equilateral,
ABC = ACB = CAB = QP R = P RQ = RQP = 60 .
Since the angles in a straight line sum to 180 , we have
180 = 65 + ABC + Y BP = 65 + 60 + Y BP . Rearranging, we have
Y BP = 180 65 60 = 55 .
Similarly, since angles in a straight line sum to 180 , we have
180 = 75 + QP R + Y P B = 75 + 60 + Y P B. Rearranging, we have
Y P B = 180 75 60 = 45 .
Since the angles in a triangle sum to 180 , in 4BY P we have
Y P B + Y BP + BY P = 180 , and so 45 + 55 + BY P = 180 . Rearranging, we have
BY P = 180 45 55 = 80 .
When two lines intersect, vertically opposite angles are equal. Since XY C and BY P are
vertically opposite angles, we have XY C = BY P = 80 .
Again, since angles in a triangle sum to 180 , in 4XY C we have
XY C + XCY + CXY = 180 . We have already found that XY C = 80 , and since
XCY = ACB, we have XCY = 60 . So, XY C + XCY + CXY = 180 becomes
80 + 60 + CXY = 180 . Rearranging, we have CXY = 180 80 60 = 40 .
Therefore, CXY = 40 .

WWW.C E M C .U WAT E R LO O.C A | T h e C E N T R E fo r E D U C AT I O N i n M AT H E M AT I C S a n d CO M P U T I N G

Problem of the Week


Problem C
A Kaleidoscope of Areas
In the diagram shown below, a 5 cm by 5 cm square is formed by arranging 25
identical 1 cm by 1 cm squares in five rows, each of which contains five squares.
The large square contains four shaded triangles.
Find the ratio of the shaded area to the unshaded area.

WWW.C E M C .U WAT E R LO O.C A | T h e C E N T R E fo r E D U C AT I O N i n M AT H E M AT I C S a n d CO M P U T I N G

Problem of the Week


Problem C and Solution
A Kaleidoscope of Areas
Problem
In the diagram, a 5 cm by 5 cm square is formed by arranging 25 identical 1
cm by 1 cm squares in five rows, each of which contains five squares. The large
square contains four shaded triangles. Find the ratio of the shaded area to the
unshaded area.
Solution

We will start by determining the area of the four shaded triangles.


We have labeled the regions A, B, C and D. We will calculate the
area of each triangle using the formula for the area of a triangle:

A
D

base height
area =
2
The triangle in region A has base 4 cm and height 3 cm.
12
2
The area of this triangle is 43
2 = 2 = 6 cm .
The triangle in region B has base 3 cm and height 2 cm.
6
2
The area of this triangle is 32
2 = 2 = 3 cm .
The triangle in region C has base 4 cm and height 2 cm.
8
2
The area of this triangle is 42
2 = 2 = 4 cm .
The triangle in region D has base 3 cm and height 3 cm.
9
2
The area of this triangle is 33
2 = 2 cm .
The total shaded area is therefore 6 + 3 + 4 +

9
2

= 13 +

9
2

26
2

9
2

35
2

cm2 .

We will now determine the area of the unshaded region. The square is 5 cm
long and 5 cm wide. Therefore, the area of the entire square is 5 5 = 25 cm2 .
The area of the unshaded region is the area of the entire square minus the area
of the shaded region. So the area of the unshaded region is
50
35
15
2
25 35
2 = 2 2 = 2 cm .
The ratioof the shaded area to the unshaded area is
35
15
2 : 2 = 35 : 15 = 7 : 3.

WWW.C E M C .U WAT E R LO O.C A | T h e C E N T R E fo r E D U C AT I O N i n M AT H E M AT I C S a n d CO M P U T I N G

Problem of the Week


Problem C
Over the Edge
A and C lie on the circumference of the circle with centre O.
OABC is a square with area 16 cm2 .
Determine the area of the shaded region, accurate to two decimal places. That
is, determine the area of the region inside square OABC and outside the circle,
accurate to two decimal places.

WWW.C E M C .U WAT E R LO O.C A | T h e C E N T R E fo r E D U C AT I O N i n M AT H E M AT I C S a n d CO M P U T I N G

Problem of the Week


Problem C and Solution
Over the Edge
Problem
A and C lie on the circumference of the circle with centre O.
OABC is a square with area 16 cm2 . Determine the area of
the shaded region, accurate to two decimal places. That is,
determine the area of the region inside square OABC and
outside the circle, accurate to two decimal places.

Solution
Since OABC is a square with area 16 cm2 , its side length must be 4 cm. That
is, OA = OC = 4 cm.
Since A lies on the circumference of the circle with centre O, the radius of the
circle is r = OA = 4 cm.
Therefore, the area of the circle is r2 = 42 = 16 cm2 .
Since OABC is a square, AOC = 90 .
1
90
=
Therefore, the area of sector OAC is
of the area of the circle.
360
4
1
In other words, the area of the sector OAC is 16 = 4 cm2 .
4
Therefore,
the area of the shaded region
= the area of the square OABC the area of the sector OAC
= 16 4
3.43 cm2 .

WWW.C E M C .U WAT E R LO O.C A | T h e C E N T R E fo r E D U C AT I O N i n M AT H E M AT I C S a n d CO M P U T I N G

Problem of the Week


Problem C
At the Fair - Intense?
A large tent is being set up for a fair.
Two poles, QU and RT , are placed perpendicular to the ground and 12 m
apart. Pole QU is 4 m in length and pole RT is 7.5 m in length.
A tarp is placed over the poles and secured to the ground at P , 3 m from the
base of pole QU , and S, 4 m from the base of pole RT .
Determine P Q + QR + RS, the length of the tarp.

R
Q
7.5

4
P 3 U

12

T 4

WWW.C E M C .U WAT E R LO O.C A | T h e C E N T R E fo r E D U C AT I O N i n M AT H E M AT I C S a n d CO M P U T I N G

Problem of the Week


Problem C and Solution
At the Fair - Intense?
Problem
A large tent is being set up for a fair. Two poles, QU
and RT , are placed perpendicular to the ground and 12
m apart. Pole QU is 4 m in length and pole RT is 7.5
m in length. A tarp is placed over the poles and secured
to the ground at P , 3 m from the base of pole QU , and
S, 4 m from the base of pole RT .

R
Q
7.5

4
P 3 U

T 4

12

Determine P Q + QR + RS, the length of the tarp.


Solution
First, we will calculate P Q and RS.
Since P U Q = 90 , we can apply the Pythagorean Theorem in 4P U Q. Thus
P Q2 = QU 2 + P U 2 = 42 + 32 = 16 + 9 = 25. Therefore, P Q = 5, since P Q > 0.
Similarly, RT S = 90 , so we can apply the Pythagorean Theorem in 4RT S.
Thus RS 2 = RT 2 + T S 2 = 7.52 + 42 = 56.25 + 16 = 72.25. Therefore,
RS = 8.5, since RS > 0.
Now we will calculate QR.
Draw a line from Q perpendicular to RT .
Let A be the point of intersection of the
perpendicular with RT . Since QA is
perpendicular to RT , QAT U is a rectangle.
Therefore, QA = U T = 12 and AT = QU = 4.
Thus AR = RT AT = 7.5 4 = 3.5.

R
3.5

Q
4
P 3 U

4
12

T 4

Since QAR = 90 , we can apply the Pythagorean Theorem in 4QAR. Thus


QR2 = QA2 + AR2 = 122 + 3.52 = 144 + 12.25 = 156.25. Therefore,
QR = 12.5, since QR > 0.
Therefore, the length of the tarp is P Q + QR + RS = 5 + 12.5 + 8.5 = 26 m.

WWW.C E M C .U WAT E R LO O.C A | T h e C E N T R E fo r E D U C AT I O N i n M AT H E M AT I C S a n d CO M P U T I N G

Problem of the Week


Problem C
Building Bigger
A student constructs a right-angled triangle, 4ABC, with an area of 6 cm2 .
She constructs a second triangle, 4DEF , whose side lengths are exactly three
times the lengths of the sides of her original triangle. That is, DE = 3AB,
EF = 3BC and DF = 3AC.
Given this information, determine the area of 4DEF .

D
A
B

It may be helpful to notice that 4ABC and 4DEF are similar. The symbol
for similarity is .
Two triangles are similar if their corresponding side lengths are in the same
ratio. That is,
If

AB
BC
AC
=
=
, then 4ABC 4DEF
DE
EF
DF

When two triangles are similar, it is also true that corresponding angles are
equal. That is,
If 4ABC 4DEF, then ABC = DEF, BCA = EF D and CAB = F DE.

WWW.C E M C .U WAT E R LO O.C A | T h e C E N T R E fo r E D U C AT I O N i n M AT H E M AT I C S a n d CO M P U T I N G

D
A
B

Problem of the Week


C

Problem C and Solution


E

Building Bigger

Problem
A student constructs a right-angled triangle, 4ABC, with an area of 6 cm2 . She constructs a
second triangle, 4DEF , whose side lengths are exactly three times the lengths of the sides of
her original triangle. That is, DE = 3AB, EF = 3BC and DF = 3AC. Given this
information, determine the area of 4DEF .

Solution

Let h represent the height of 4ABC and b represent the base


of 4ABC.
, we have 6 = bh
, or
Using the formula area = baseheight
2
2
b h = 12.

4DEF is formed by multiplying each of the original side lengths by 3. Since the ratio of the
AB
AC
side lengths does not change, that is, DE
= BC
= DF
= 13 , 4ABC 4DEF .
EF
Since corresponding angles in similar triangles are equal,
DEF = ABC = 90 , and so 4DEF is also right-angled.
Since 4ABC has height h, 4DEF has height 3 h.
Since 4ABC has base b, 4DEF has base 3 b.
Using the formula area =

baseheight
,
2

(3 b) (3 h)
2
9bh
=
2
9 12
=
, since b h = 12
2
= 54 cm2

D
3h

3b

the area of 4DEF =

As an extension:
Notice that 4DEF has side lengths that are each three times the corresponding lengths of
4ABC and that the area of the 4DEF ended up being 54 = 9 6 = 32 area of 4ABC.
This is not a coincidence. It turns out that if 4ABC 4DEF and the side lengths of
4DEF are all k times the corresponding side lengths of 4ABC, then
area of 4DEF = k 2 area of 4ABC
Can you show that this is always true for a right-angled triangle? Can you show that this is
always true for any triangle?

WWW.C E M C .U WAT E R LO O.C A | T h e C E N T R E fo r E D U C AT I O N i n M AT H E M AT I C S a n d CO M P U T I N G

Problem of the Week


Problem C
A Fair Split
The 6 m by 6 m square shown below is to be divided into three equal areas
using two cuts. One possible way for you to cut the square would be to make a
horizontal slice through H and a second horizontal slice through K. This
method of cutting the square works but is not very creative.
To make things a little more interesting, you must still make two straight cuts
but each cut must start at point P . Each of these two cuts will pass through a
point on the outside of the square. Which other labelled points will the cuts
pass through?

U V

W Q
G
H
J
K
L

WWW.C E M C .U WAT E R LO O.C A | T h e C E N T R E fo r E D U C AT I O N i n M AT H E M AT I C S a n d CO M P U T I N G

U V

W Q
G
H
J
K
L

Problem of the Week


Problem C and Solution
A Fair Split

Problem
The 6 m by 6 m square shown above is to be divided into three equal areas using two cuts.
You must make two straight cuts but each cut must start at point P . Each of these two cuts
will pass through a point on the outside of the square. Which other labelled points will the
cuts pass through?

Solution
The area of the entire 6 m by 6 m square is 6 6 = 36 m2 . Since the square is divided into
36
= 12 m2 .
three regions of equal area, the area of each region must be
3
Consider the line through P that passes through some point on
the vertical line segment QM . Let A be the point where this line
intersects QM . Since P M Q = 90 , 4P M A is a right triangle with
base P M = 6 m and height M A.
base height
Using the formula area =
, we have
2
6
P
6 MA
area of 4P M A =
= 3 M A.
2
We require that the area of 4P M A = 12 m2 . Therefore, 3 M A = 12 and so M A = 4 m.
Since H is the point on QM with M H = 4 m, we must have A = H.
R
B
Therefore, one line passes through the point H.
Consider the line through P that passes through some point on the
horizontal line segment RQ. Let B be the point where this line
intersects RQ. Since P RQ = 90 , 4P RB is a right triangle with
height P R = 6 m and base RB.
base height
Using the formula area =
, we have
2
RB 6
= 3 RB.
area of 4P RB =
2

We require that the area of 4P RB = 12 m2 . Therefore, 3 RB = 12, and so RB = 4 m.


Since V is the point on RQ with RV = 4 m, we must have B = V .
Therefore, the other line passes through the point V .
Therefore, one line passes through the point H and the other passes through the point V .

Q
A

WWW.C E M C .U WAT E R LO O.C A | T h e C E N T R E fo r E D U C AT I O N i n M AT H E M AT I C S a n d CO M P U T I N G

Problem of the Week


Problem C
Prepare for the Cold
A rectangular container with base 9 cm by 11 cm has a height of 38.5 cm.
Assuming that water expands 10% when it freezes, determine the depth to
which the container can be filled so that when the contents freeze, the ice does
not go above the top of the container.

WWW.C E M C .U WAT E R LO O.C A | T h e C E N T R E fo r E D U C AT I O N i n M AT H E M AT I C S a n d CO M P U T I N G

Problem of the Week


Problem C and Solutions
Prepare for the Cold

Problem
A rectangular container with base 9 cm by 11 cm has a height of 38.5 cm. Assuming that
water expands 10% when it freezes, determine the depth to which the container can be filled so
that when the contents freeze, the ice does not go above the top of the container.

Solution 1
To determine the volume of a rectangular solid, multiply the length, width and height. So the
maximum volume of the container will be
9 11 38.5 = 3811.5 cm3 .
Let the original depth of water in the container be h cm.
Then the water volume before freezing is 9 11 h = (99 h) cm3 . After the water freezes, the
volume increases by 10% to 110% of its current volume. So after freezing, the volume will be
110% of 99 h = 1.1 99 h = (108.9 h) cm3 .
But the volume after freezing is the maximum volume, 3811.5 cm3 . Therefore,
108.9 h = 3811.5 and it follows that h = 3811.5 108.9 = 35 cm.
The container can be filled with water to a depth of 35 cm so that when it freezes the ice will
not go over the top of the container.

Solution 2
In this solution we note that the length and width remain the same in both the volume
calculation before and after the water freezes. We need only concern ourselves with the change
in the depth of the water.
Let the original depth of water in the container be h cm.
After freezing, the depth increases by 10% to 110% of its depth before freezing. So after
freezing, the depth will be 110% of h = 1.1 h = 38.5 cm, the maximum height of the
container. Then h = 38.5 1.1 = 35 cm.
The container can be filled with water to a depth of 35 cm so that when it freezes the ice will
not go over the top of the container.

WWW.C E M C .U WAT E R LO O.C A | T h e C E N T R E fo r E D U C AT I O N i n M AT H E M AT I C S a n d CO M P U T I N G

Problem of the Week


Problem C
A Kaleidoscope of Areas
In the diagram shown below, a 5 cm by 5 cm square is formed by arranging 25
identical 1 cm by 1 cm squares in five rows, each of which contains five squares.
The large square contains four shaded triangles.
Find the ratio of the shaded area to the unshaded area.

WWW.C E M C .U WAT E R LO O.C A | T h e C E N T R E fo r E D U C AT I O N i n M AT H E M AT I C S a n d CO M P U T I N G

Problem of the Week


Problem C and Solution
A Kaleidoscope of Areas
Problem
In the diagram, a 5 cm by 5 cm square is formed by arranging 25 identical 1
cm by 1 cm squares in five rows, each of which contains five squares. The large
square contains four shaded triangles. Find the ratio of the shaded area to the
unshaded area.
Solution

We will start by determining the area of the four shaded triangles.


We have labeled the regions A, B, C and D. We will calculate the
area of each triangle using the formula for the area of a triangle:

A
D

base height
area =
2
The triangle in region A has base 4 cm and height 3 cm.
12
2
The area of this triangle is 43
2 = 2 = 6 cm .
The triangle in region B has base 3 cm and height 2 cm.
6
2
The area of this triangle is 32
2 = 2 = 3 cm .
The triangle in region C has base 4 cm and height 2 cm.
8
2
The area of this triangle is 42
2 = 2 = 4 cm .
The triangle in region D has base 3 cm and height 3 cm.
9
2
The area of this triangle is 33
2 = 2 cm .
The total shaded area is therefore 6 + 3 + 4 +

9
2

= 13 +

9
2

26
2

9
2

35
2

cm2 .

We will now determine the area of the unshaded region. The square is 5 cm
long and 5 cm wide. Therefore, the area of the entire square is 5 5 = 25 cm2 .
The area of the unshaded region is the area of the entire square minus the area
of the shaded region. So the area of the unshaded region is
50
35
15
2
25 35
2 = 2 2 = 2 cm .
The ratioof the shaded area to the unshaded area is
35
15
2 : 2 = 35 : 15 = 7 : 3.

WWW.C E M C .U WAT E R LO O.C A | T h e C E N T R E fo r E D U C AT I O N i n M AT H E M AT I C S a n d CO M P U T I N G

Problem of the Week


Problem C
Over the Edge
A and C lie on the circumference of the circle with centre O.
OABC is a square with area 16 cm2 .
Determine the area of the shaded region, accurate to two decimal places. That
is, determine the area of the region inside square OABC and outside the circle,
accurate to two decimal places.

WWW.C E M C .U WAT E R LO O.C A | T h e C E N T R E fo r E D U C AT I O N i n M AT H E M AT I C S a n d CO M P U T I N G

Problem of the Week


Problem C and Solution
Over the Edge
Problem
A and C lie on the circumference of the circle with centre O.
OABC is a square with area 16 cm2 . Determine the area of
the shaded region, accurate to two decimal places. That is,
determine the area of the region inside square OABC and
outside the circle, accurate to two decimal places.

Solution
Since OABC is a square with area 16 cm2 , its side length must be 4 cm. That
is, OA = OC = 4 cm.
Since A lies on the circumference of the circle with centre O, the radius of the
circle is r = OA = 4 cm.
Therefore, the area of the circle is r2 = 42 = 16 cm2 .
Since OABC is a square, AOC = 90 .
1
90
=
Therefore, the area of sector OAC is
of the area of the circle.
360
4
1
In other words, the area of the sector OAC is 16 = 4 cm2 .
4
Therefore,
the area of the shaded region
= the area of the square OABC the area of the sector OAC
= 16 4
3.43 cm2 .

WWW.C E M C .U WAT E R LO O.C A | T h e C E N T R E fo r E D U C AT I O N i n M AT H E M AT I C S a n d CO M P U T I N G

Problem of the Week


Problem C
Framed
Four rectangular strips of wood, each 30 cm long and 3 cm wide, are arranged
to form the outer section of a picture frame, as shown.
Determine the area inside the wooden frame.

WWW.C E M C .U WAT E R LO O.C A | T h e C E N T R E fo r E D U C AT I O N i n M AT H E M AT I C S a n d CO M P U T I N G

Problem of the Week


Problem C and Solutions
Framed
Problem
Four rectangular strips of wood, each 30 cm long and 3 cm wide, are arranged
to form the outer section of a picture frame, as shown. Determine the area
inside the wooden frame.

Solution 1
The inner square has side length 30 3 = 27 cm.
Area = Length Width = 27 27 = 729 cm2 .

30

27

Therefore, the enclosed area is 729 cm2 .


3

Solution 2

Each rectangle has area 30 3 = 90 cm2 .


The outer square has a side length equal to
30 + 3 = 33 cm.

33

30

The area of the outer square is therefore


33 33 = 1089 cm2 .
The area of the inner square is equal to the area
of the outer square minus the areas of the four
rectangles.

30

Therefore, the area of the inner square is 1089 4 90 = 729 cm2 .

33

WWW.C E M C .U WAT E R LO O.C A | T h e C E N T R E fo r E D U C AT I O N i n M AT H E M AT I C S a n d CO M P U T I N G

Problem of the Week


Problem C
At the Fair - Intense?
A large tent is being set up for a fair.
Two poles, QU and RT , are placed perpendicular to the ground and 12 m
apart. Pole QU is 4 m in length and pole RT is 7.5 m in length.
A tarp is placed over the poles and secured to the ground at P , 3 m from the
base of pole QU , and S, 4 m from the base of pole RT .
Determine P Q + QR + RS, the length of the tarp.

R
Q
7.5

4
P 3 U

12

T 4

WWW.C E M C .U WAT E R LO O.C A | T h e C E N T R E fo r E D U C AT I O N i n M AT H E M AT I C S a n d CO M P U T I N G

Problem of the Week


Problem C and Solution
At the Fair - Intense?
Problem
A large tent is being set up for a fair. Two poles, QU
and RT , are placed perpendicular to the ground and 12
m apart. Pole QU is 4 m in length and pole RT is 7.5
m in length. A tarp is placed over the poles and secured
to the ground at P , 3 m from the base of pole QU , and
S, 4 m from the base of pole RT .

R
Q
7.5

4
P 3 U

T 4

12

Determine P Q + QR + RS, the length of the tarp.


Solution
First, we will calculate P Q and RS.
Since P U Q = 90 , we can apply the Pythagorean Theorem in 4P U Q. Thus
P Q2 = QU 2 + P U 2 = 42 + 32 = 16 + 9 = 25. Therefore, P Q = 5, since P Q > 0.
Similarly, RT S = 90 , so we can apply the Pythagorean Theorem in 4RT S.
Thus RS 2 = RT 2 + T S 2 = 7.52 + 42 = 56.25 + 16 = 72.25. Therefore,
RS = 8.5, since RS > 0.
Now we will calculate QR.
Draw a line from Q perpendicular to RT .
Let A be the point of intersection of the
perpendicular with RT . Since QA is
perpendicular to RT , QAT U is a rectangle.
Therefore, QA = U T = 12 and AT = QU = 4.
Thus AR = RT AT = 7.5 4 = 3.5.

R
3.5

Q
4
P 3 U

4
12

T 4

Since QAR = 90 , we can apply the Pythagorean Theorem in 4QAR. Thus


QR2 = QA2 + AR2 = 122 + 3.52 = 144 + 12.25 = 156.25. Therefore,
QR = 12.5, since QR > 0.
Therefore, the length of the tarp is P Q + QR + RS = 5 + 12.5 + 8.5 = 26 m.

WWW.C E M C .U WAT E R LO O.C A | T h e C E N T R E fo r E D U C AT I O N i n M AT H E M AT I C S a n d CO M P U T I N G

Problem of the Week


Problem C
Building Bigger
A student constructs a right-angled triangle, 4ABC, with an area of 6 cm2 .
She constructs a second triangle, 4DEF , whose side lengths are exactly three
times the lengths of the sides of her original triangle. That is, DE = 3AB,
EF = 3BC and DF = 3AC.
Given this information, determine the area of 4DEF .

D
A
B

It may be helpful to notice that 4ABC and 4DEF are similar. The symbol
for similarity is .
Two triangles are similar if their corresponding side lengths are in the same
ratio. That is,
If

AB
BC
AC
=
=
, then 4ABC 4DEF
DE
EF
DF

When two triangles are similar, it is also true that corresponding angles are
equal. That is,
If 4ABC 4DEF, then ABC = DEF, BCA = EF D and CAB = F DE.

WWW.C E M C .U WAT E R LO O.C A | T h e C E N T R E fo r E D U C AT I O N i n M AT H E M AT I C S a n d CO M P U T I N G

D
A
B

Problem of the Week


C

Problem C and Solution


E

Building Bigger

Problem
A student constructs a right-angled triangle, 4ABC, with an area of 6 cm2 . She constructs a
second triangle, 4DEF , whose side lengths are exactly three times the lengths of the sides of
her original triangle. That is, DE = 3AB, EF = 3BC and DF = 3AC. Given this
information, determine the area of 4DEF .

Solution

Let h represent the height of 4ABC and b represent the base


of 4ABC.
, we have 6 = bh
, or
Using the formula area = baseheight
2
2
b h = 12.

4DEF is formed by multiplying each of the original side lengths by 3. Since the ratio of the
AB
AC
side lengths does not change, that is, DE
= BC
= DF
= 13 , 4ABC 4DEF .
EF
Since corresponding angles in similar triangles are equal,
DEF = ABC = 90 , and so 4DEF is also right-angled.
Since 4ABC has height h, 4DEF has height 3 h.
Since 4ABC has base b, 4DEF has base 3 b.
Using the formula area =

baseheight
,
2

(3 b) (3 h)
2
9bh
=
2
9 12
=
, since b h = 12
2
= 54 cm2

D
3h

3b

the area of 4DEF =

As an extension:
Notice that 4DEF has side lengths that are each three times the corresponding lengths of
4ABC and that the area of the 4DEF ended up being 54 = 9 6 = 32 area of 4ABC.
This is not a coincidence. It turns out that if 4ABC 4DEF and the side lengths of
4DEF are all k times the corresponding side lengths of 4ABC, then
area of 4DEF = k 2 area of 4ABC
Can you show that this is always true for a right-angled triangle? Can you show that this is
always true for any triangle?

WWW.C E M C .U WAT E R LO O.C A | T h e C E N T R E fo r E D U C AT I O N i n M AT H E M AT I C S a n d CO M P U T I N G

Problem of the Week


Problem C
A Fair Split
The 6 m by 6 m square shown below is to be divided into three equal areas
using two cuts. One possible way for you to cut the square would be to make a
horizontal slice through H and a second horizontal slice through K. This
method of cutting the square works but is not very creative.
To make things a little more interesting, you must still make two straight cuts
but each cut must start at point P . Each of these two cuts will pass through a
point on the outside of the square. Which other labelled points will the cuts
pass through?

U V

W Q
G
H
J
K
L

WWW.C E M C .U WAT E R LO O.C A | T h e C E N T R E fo r E D U C AT I O N i n M AT H E M AT I C S a n d CO M P U T I N G

U V

W Q
G
H
J
K
L

Problem of the Week


Problem C and Solution
A Fair Split

Problem
The 6 m by 6 m square shown above is to be divided into three equal areas using two cuts.
You must make two straight cuts but each cut must start at point P . Each of these two cuts
will pass through a point on the outside of the square. Which other labelled points will the
cuts pass through?

Solution
The area of the entire 6 m by 6 m square is 6 6 = 36 m2 . Since the square is divided into
36
= 12 m2 .
three regions of equal area, the area of each region must be
3
Consider the line through P that passes through some point on
the vertical line segment QM . Let A be the point where this line
intersects QM . Since P M Q = 90 , 4P M A is a right triangle with
base P M = 6 m and height M A.
base height
Using the formula area =
, we have
2
6
P
6 MA
area of 4P M A =
= 3 M A.
2
We require that the area of 4P M A = 12 m2 . Therefore, 3 M A = 12 and so M A = 4 m.
Since H is the point on QM with M H = 4 m, we must have A = H.
R
B
Therefore, one line passes through the point H.
Consider the line through P that passes through some point on the
horizontal line segment RQ. Let B be the point where this line
intersects RQ. Since P RQ = 90 , 4P RB is a right triangle with
height P R = 6 m and base RB.
base height
Using the formula area =
, we have
2
RB 6
= 3 RB.
area of 4P RB =
2

We require that the area of 4P RB = 12 m2 . Therefore, 3 RB = 12, and so RB = 4 m.


Since V is the point on RQ with RV = 4 m, we must have B = V .
Therefore, the other line passes through the point V .
Therefore, one line passes through the point H and the other passes through the point V .

Q
A

WWW.C E M C .U WAT E R LO O.C A | T h e C E N T R E fo r E D U C AT I O N i n M AT H E M AT I C S a n d CO M P U T I N G

Problem of the Week


Problem C
The First of Three Average Problems
In football, the player who kicks the ball is referred to as the punter. During a
recent football game, the punter, Khan Bootit, kicked the ball five times. His
longest kick was 44 yards and he averaged 35 yards per kick. Each of his kicks
was a different positive integer length.
Determine the minimum possible length of Khans shortest kick.

WWW.C E M C .U WAT E R LO O.C A | T h e C E N T R E fo r E D U C AT I O N i n M AT H E M AT I C S a n d CO M P U T I N G

Problem of the Week


Problem C and Solution
The First of Three Average Problems
Problem
In football, the player who kicks the ball is referred to as the punter. During a
recent football game, the punter, Khan Kickit, kicked the ball five times. His
longest kick was 44 yards and he averaged 35 yards per kick. Each of his kicks
was a different positive integer length. Determine the minimum possible length
of Khans shortest kick.
Solution
To determine an average, determine the sum of the numbers in the set and
divide by the number of numbers in the set. It follows that the sum of the
numbers in a set is the average times the number of numbers in the set.
Since Khans average was 35 yards per kick and he kicked the ball five times,
the total length of all his kicks was 5 35 = 175 yards. His longest kick was 44
yards, so his remaining kicks covered a total of 175 44 = 131 yards.
To find the minimum length of his shortest kick make the other three kicks as
long as possible. Each of these kicks must be a different integer length less
than 44 yards long. So for the three kicks to be as long as possible, they must
be 43 yards, 42 yards, and 41 yards, respectively.
The minimum length of the shortest kick is therefore 131 43 42 41 = 5
yards.
If he had been able to have different kicks with the same integer length, then
the minimum length of his shortest kick would be 175 4 44 = 1 yard. (It
would have travelled backwards.) But the kicks all had to be positive integer
lengths so his minimum shortest kick would have been 1 yard with his other
kicks being such that the total yards travelled by all of his other four kicks
would be 174 yards. There would be a few different possible kick lengths that
would make this possible. For example, his five kicks could be 1, 44, 44, 44,
and 42 yards each, respectively or 1, 44, 44, 43, and 43 yards each, respectively.

WWW.C E M C .U WAT E R LO O.C A | T h e C E N T R E fo r E D U C AT I O N i n M AT H E M AT I C S a n d CO M P U T I N G

Problem of the Week


Problem C
Prepare for the Cold
A rectangular container with base 9 cm by 11 cm has a height of 38.5 cm.
Assuming that water expands 10% when it freezes, determine the depth to
which the container can be filled so that when the contents freeze, the ice does
not go above the top of the container.

WWW.C E M C .U WAT E R LO O.C A | T h e C E N T R E fo r E D U C AT I O N i n M AT H E M AT I C S a n d CO M P U T I N G

Problem of the Week


Problem C and Solutions
Prepare for the Cold

Problem
A rectangular container with base 9 cm by 11 cm has a height of 38.5 cm. Assuming that
water expands 10% when it freezes, determine the depth to which the container can be filled so
that when the contents freeze, the ice does not go above the top of the container.

Solution 1
To determine the volume of a rectangular solid, multiply the length, width and height. So the
maximum volume of the container will be
9 11 38.5 = 3811.5 cm3 .
Let the original depth of water in the container be h cm.
Then the water volume before freezing is 9 11 h = (99 h) cm3 . After the water freezes, the
volume increases by 10% to 110% of its current volume. So after freezing, the volume will be
110% of 99 h = 1.1 99 h = (108.9 h) cm3 .
But the volume after freezing is the maximum volume, 3811.5 cm3 . Therefore,
108.9 h = 3811.5 and it follows that h = 3811.5 108.9 = 35 cm.
The container can be filled with water to a depth of 35 cm so that when it freezes the ice will
not go over the top of the container.

Solution 2
In this solution we note that the length and width remain the same in both the volume
calculation before and after the water freezes. We need only concern ourselves with the change
in the depth of the water.
Let the original depth of water in the container be h cm.
After freezing, the depth increases by 10% to 110% of its depth before freezing. So after
freezing, the depth will be 110% of h = 1.1 h = 38.5 cm, the maximum height of the
container. Then h = 38.5 1.1 = 35 cm.
The container can be filled with water to a depth of 35 cm so that when it freezes the ice will
not go over the top of the container.

WWW.C E M C .U WAT E R LO O.C A | T h e C E N T R E fo r E D U C AT I O N i n M AT H E M AT I C S a n d CO M P U T I N G

Problem of the Week


Problem C
Mmmm ... Chocolate
A hockey team is selling chocolate bars as a fundraiser. Only one type of
chocolate bar is being sold, and each bar sells for $2.70. Amanda, Ben, Cole
and David pool their money together to buy a total of 5 chocolate bars.
Amanda paid $8.10, Ben paid $2.70 and Cole paid $1.80. David paid the
remainder. They divide the chocolate bars so that the fraction of the total that
each person paid is equal to the fraction of the total of chocolate that each
person receives.
How much chocolate should each person receive?

WWW.C E M C .U WAT E R LO O.C A | T h e C E N T R E fo r E D U C AT I O N i n M AT H E M AT I C S a n d CO M P U T I N G

Problem of the Week


Problem C and Solution
Mmmm ... Chocolate
Problem
A hockey team is selling chocolate bars as a fundraiser. Only one type of
chocolate bar is being sold, and each bar sells for $2.70. Amanda, Ben, Cole
and David pool their money together to buy a total of 5 chocolate bars.
Amanda paid $8.10, Ben paid $2.70 and Cole paid $1.80. David paid the
remainder. They divide the chocolate bars so that the fraction of the total that
each person paid is equal to the fraction of the total of chocolate that each
person receives. How much chocolate should each person receive?
Solution
If Amanda, Ben, Cole and David buy a total of 5 chocolate bars at $2.70 per
bar, then they paid a total of 5 $2.70 = $13.50.
Amanda paid $8.10, Ben paid $2.70, Cole paid $1.80 and David paid the
remainder. Therefore, David paid $13.50 $8.10 $2.70 $1.80 = $0.90.
8.10
81
Amanda should receive 13.50
= 135
=
So Amanda should receive 35 of 5, or
2.70
27
Ben should receive 13.50
= 135
=
So Ben should receive 15 of 5, or

1
5
1
5

3
5
3
5

of the total amount of chocolate.


5 = 3 chocolate bars.

of the total amount of chocolate.


5 = 1 chocolate bar.

1.80
18
2
Cole should receive 13.50
= 135
= 15
of the total amount of chocolate.
2
2
So Cole should receive 15
of 5, or 15
5 = 23 of a chocolate bar.
0.90
9
1
David should receive 13.50
= 135
= 15
of the total amount of chocolate.
1
1
So David should receive 15
of 5, or 15
5 = 13 of a chocolate bar.

We should check that all of the chocolate has been distributed. If there were
any errors in our solution, this may help us catch them. Amanda receives 3
chocolate bars, Ben receives 1 chocolate bar, Cole receives 23 of a chocolate bar
and David receives 13 of a chocolate bar. The total number of chocolate bars
distributed is 3 + 1 + 32 + 13 = 5, as required.

WWW.C E M C .U WAT E R LO O.C A | T h e C E N T R E fo r E D U C AT I O N i n M AT H E M AT I C S a n d CO M P U T I N G

Problem of the Week


Problem C
Looking for Nothing
The product of the positive integers 1 to 6 is
6 5 4 3 2 1 = 720
and can be written in an abbreviated form as 6!. We say, 6 factorial . So
6! = 720.
The product of the positive integers from 1 to 12 is
12 11 10 . . . 3 2 1 = 479 001 600
and can be written in an abbreviated form as 12!. We say, 12 factorial .
The represents the product of all of the missing integers between 10 and 3.
For a positive integer n, the product of the positive integers from 1 to n is n!.
Find the smallest possible value of n such that n! ends in exactly six zeroes.

WWW.C E M C .U WAT E R LO O.C A | T h e C E N T R E fo r E D U C AT I O N i n M AT H E M AT I C S a n d CO M P U T I N G

Problem of the Week


Problem C and Solution
Looking for Nothing
Problem
The product of the positive integers 1 to 6 is 6 5 4 3 2 1 = 720. and
can be written in an abbreviated form as 6!. We say, 6 factorial . So 6! = 720.
The product of the positive integers from 1 to 12 is
12 11 10 . . . 3 2 1 = 479 001 600
and can be written in an abbreviated form as 12!. We say, 12 factorial .
The represents the product of all of the missing integers between 10 and 3.
For a positive integer n, the product of the positive integers from 1 to n is n!.
Find the smallest possible value of n such that n! ends in exactly six zeroes.
Solution
We start by examining the first few factorials:
1! = 1
2! = 2 1 = 2
3! = 3 2 1 = 6
4! = 4 3 2 1 = 24
5! = 5 4 3 2 1 = 120
6! = 6 (5 4 3 2 1) = 6 5! = 6(120) = 720
7! = 7 (6 5 4 3 2 1) = 7 6! = 7(720) = 5040
8! = 8 (7 6 5 4 3 2 1) = 8 7! = 8(5040) = 40 320
9! = 9 (8 7 6 5 4 3 2 1) = 9 8! = 9(40 320) = 362 880
10! = 10 (9 8 7 6 5 4 3 2 1) = 10 9! = 10(362 880) = 3 628 800
These numbers are getting very large and soon will not fit on the display of a
standard calculator. So, lets look at what is going on.

WWW.C E M C .U WAT E R LO O.C A | T h e C E N T R E fo r E D U C AT I O N i n M AT H E M AT I C S a n d CO M P U T I N G

We observe that 5! ends in 0 and 10! ends in 00. Notice that the number of
zeros at the end of the number increased by one at each of 5! and at 10!. Why
is this?
A zero is added to the end of a number when we multiply by 10. Multiplying a
number by 10 is the same as multiplying a number by 2 and then by 5, or by 5
and then by 2, since 2 5 = 10 and 5 2 = 10. We must determine the next
time we multiply by 2 and 5 (in some order), to know the next time the
number of zeros at the end of the number increases again. Every time we
multiply by an even number we are multiplying by at least one more 2. There
are less multiples of five. Each multiple of five will affect the number of zeroes
at the end of the product.
Multiplying by 11, 12, 13, and 14 increases the number of 2s we multiply by
but not the number of 5s. So the number of zeros at the end of the product
does not change. The next time we multiply by a 5 is when we multiply by 15
since 15 = 5 3. So 15! will end in exactly three zeroes, 000.
Multiplying by 16, 17, 18, and 19 increases the number of 2s we multiply by
but not the number of 5s. So the number of zeros at the end of the product
does not change. The next time we multiply by a 5 is when we multiply by 20
since 20 = 4 5. So 20! will end in exactly four zeroes, 0000.
Multiplying by 21, 22, 23, and 24 increases the number of 2s we multiply by
but not the number of 5s. The next time we multiply by a 5 is when we
multiply by 25. In fact, multiplying by 25 is the same as multiplying by 5 twice
since 25 = 5 5. So when we multiply by 25, we will increase the number of
zeros on the end of the product by two. So 25! will end in exactly six zeroes,
000 000.
Then, for n! to end in six zeroes, the smallest value of n is 25. That is, 25! is
the smallest factorial that ends in exactly six zeroes. (It could be noted that
26!, 27!, 28!, and 29! also end in six zeroes.)

WWW.C E M C .U WAT E R LO O.C A | T h e C E N T R E fo r E D U C AT I O N i n M AT H E M AT I C S a n d CO M P U T I N G

Problem of the Week


Problem C
Factors to Consider
Two different positive whole numbers, a and b, are both greater than 1. The
second number, b, is larger than the first number. After the second number is
increased by 1, it is multiplied by the first number. The resulting product is
260. This information can be represented by the equation a (b + 1) = 260.
Determine all possible pairs of numbers that satisfy the equation.

Pairs?

WWW.C E M C .U WAT E R LO O.C A | T h e C E N T R E fo r E D U C AT I O N i n M AT H E M AT I C S a n d CO M P U T I N G

Problem of the Week


Problem C and Solution
Factors to Consider
Problem
Two different positive whole numbers, a and b, are both greater than 1. The second number, b,
is larger than the first number. After the second number is increased by 1, it is multiplied by
the first number. The resulting product is 260. This information can be represented by the
equation a (b + 1) = 260.
Determine all possible pairs of numbers that satisfy the equation.

Solution
In considering only the equation a (b + 1) = 260, we see that we are looking for two whole
numbers, each greater than 1, that multiply to 260. The first number is a and the second
number is a number that is 1 greater than b. We also want the second number to be greater
than the first number.
We can generate the following list of valid pairs of numbers that multiply to 260:
260 = 2 130 = 4 65 = 5 52 = 10 26 = 13 20.
There are other pairs of positive numbers that multiply to 260 but each of them can be
excluded because of the given restrictions.
We can exclude 260 = 1 260 and 260 = 260 1 because both numbers must be greater than 1.
We can also exclude 260 = 130 2, 260 = 65 4, 260 = 52 5, 260 = 26 10, and
260 = 20 13 because the second number in the product must be larger than the first number.
Since b is 1 less than the second number in each valid product, we can generate the pairs of
whole numbers that satisfy the equation a (b + 1) = 260. We will write the number pairs as
ordered pairs in the form (a, b).
The
The
The
The
The

first ordered pair is (2, 129) since 2 (129 + 1) = 2 130 = 260.


second ordered pair is (4, 64) since 4 (64 + 1) = 4 65 = 260.
third ordered pair is (5, 51) since 5 (51 + 1) = 5 52 = 260.
fourth ordered pair is (10, 25) since 10 (25 + 1) = 10 26 = 260.
final ordered pair is (13, 19) since 13 (19 + 1) = 13 20 = 260.

The five ordered pairs (a, b) that satisfy the equation a (b + 1) = 260 as well as all the other
conditions are (2, 129), (4, 64), (5, 51), (10, 25), and (13, 19).

WWW.C E M C .U WAT E R LO O.C A | T h e C E N T R E fo r E D U C AT I O N i n M AT H E M AT I C S a n d CO M P U T I N G

Problem of the Week


Problem C
Blue Jays versus Orioles
There are 15 Blue Jays and 14 Orioles that wish to rest on the branches of
three older trees.
Each of the trees will have at least 4 Blue Jays and 2 Orioles. However, no tree
may have more Orioles than Blue Jays in its branches.
Determine the largest number of birds that can be in one tree.

WWW.C E M C .U WAT E R LO O.C A | T h e C E N T R E fo r E D U C AT I O N i n M AT H E M AT I C S a n d CO M P U T I N G

Problem of the Week


Problem C and Solution
Blue Jays versus Orioles
Problem
There are 15 Blue Jays and 14 Orioles that wish to rest on the branches of
three older trees. Each of the trees will have at least 4 Blue Jays and 2 Orioles.
However, no tree may have more Orioles than Blue Jays in its branches.
Determine the largest number of birds that can be in one tree.
Solution
Since each tree contains at least 4 Blue Jays and 2 Orioles, lets start by
putting this minimum number of Blue Jays and Orioles in each tree.
Tree 1
Tree 2
Tree 3
4 Blue Jays, 2 Orioles 4 Blue Jays, 2 Orioles 4 Blue Jays, 2 Orioles
The number of Blue Jays not yet in a tree is 15 4 4 4 = 3.
The number of Orioles not yet in a tree is 14 2 2 2 = 8.
To produce the greatest number in a tree, as many as possible of the remaining
birds should be put in one particular tree. Lets start by putting all of the
remaining Blue Jays in Tree 1. Then we have
Tree 1
Tree 2
Tree 3
7 Blue Jays, 2 Orioles 4 Blue Jays, 2 Orioles 4 Blue Jays, 2 Orioles
Lets put as many Orioles in Tree 1 as possible. Since Tree 1 cannot have more
Orioles than Blue Jays, we can put at most 5 more Orioles in Tree 1. Now we
have
Tree 1
Tree 2
Tree 3
7 Blue Jays, 7 Orioles 4 Blue Jays, 2 Orioles 4 Blue Jays, 2 Orioles
The number of Orioles that are still not in a tree is 14 7 2 2 = 3.
We cannot place any of these Orioles in Tree 1 because then there will be more
Orioles than Blue Jays in that tree. Can we place these in the remaining two
trees? We can place 2 in Tree 2 and 1 in Tree 3. So we have
Tree 1
Tree 2
Tree 3
7 Blue Jays, 7 Orioles 4 Blue Jays, 4 Orioles 4 Blue Jays, 3 Orioles
Therefore, the largest number of birds that can be in one tree is 14.

WWW.C E M C .U WAT E R LO O.C A | T h e C E N T R E fo r E D U C AT I O N i n M AT H E M AT I C S a n d CO M P U T I N G

Problem of the Week


Problem C
All Dried Up
The water content of an apricot, by weight, is 80%. Therefore, 20% of the
apricot, by weight, is other material.
When left in the sun to dry, an apricot loses 75% of its water content, and
none of the other material.
What percent of a dried apricot is water?

WWW.C E M C .U WAT E R LO O.C A | T h e C E N T R E fo r E D U C AT I O N i n M AT H E M AT I C S a n d CO M P U T I N G

Problem of the Week


Problem C and Solutions
All Dried Up
Problem
The water content of an apricot, by weight, is 80%. Therefore, 20% of the
apricot, by weight, is other material. When left in the sun to dry, an apricot
loses 75% of its water content, and none of the other material. What percent of
a dried apricot is water?
Solution 1
Lets consider an apricot that originally weighs 100 g. If 80% of the weight is
water, that means that 80 g is water and 20 g is other material.
When left in the sun, the apricot loses 75% of its water weight. So it loses
75% of 80 g = 0.75 80 = 60 g of water, and 80 60 = 20 g of water remains.
The dried apricot is still 20 g of other material. Therefore, the dried apricot
consists of 20 g of water and 20 g other material.
20
The dried apricot is therefore
100% = 50% water.
20 + 20
Solution 2
Suppose the apricot originally weighs x g. If 80% of the weight is water, that
means that 80% of x = 0.8 x = 0.8x is water and 20% of x = 0.2x is other
material.
When left in the sun, the apricot loses 75% of its water weight. So it loses 75%
of 0.8x = 0.75 0.8x = 0.6x of water, and therefore 0.8x 0.6x = 0.2x of
water remains.
The dried apricot is still 0.2x of other material. Therefore, the dried apricot
consists of 0.2x g of water and 0.2x g other material.
Since the amount of water in the dried apricot is the same as the amount of
other material, the dried apricot consists of 50% water and 50% other material.

WWW.C E M C .U WAT E R LO O.C A | T h e C E N T R E fo r E D U C AT I O N i n M AT H E M AT I C S a n d CO M P U T I N G

Problem of the Week


Problem C
Neighbours
The houses on Consecutive Crescent are arranged so that each house is directly
opposite another house.
The houses are numbered consecutively 1, 2, 3, . . . and so on along one side.
Once the end of that side of the crescent is reached, the consecutive numbering
continues at the house on the other side of the street opposite house number 1.
The consecutive numbering continues along this second side until the last
house is numbered.
If there were ten houses on the crescent, they would be numbered as shown in
the diagram below.
However, on the actual crescent, when the residents of house number 37 look
directly across the street they see house number 84. How many houses are on
Consecutive Crescent?

CONSECUTIVE CRESCENT

10

WWW.C E M C .U WAT E R LO O.C A | T h e C E N T R E fo r E D U C AT I O N i n M AT H E M AT I C S a n d CO M P U T I N G

Problem of the Week


Problem C and Solutions
Neighbours
Problem
The houses on Consecutive Crescent are arranged so that each house is directly opposite
another house. The houses are numbered consecutively 1, 2, 3, . . . and so on along one side.
Once the end of that side of the crescent is reached, the consecutive numbering continues at
the house on the other side of the street opposite house number 1. The consecutive numbering
continues along this second side until the last house is numbered. When the residents of house
number 37 look directly across the street they see house number 84. How many houses are on
Consecutive Crescent?

Solution 1
In this solution we will use logic to reason the number of houses on Consecutive Crescent.
There are 36 houses before house 37 on the one side of the road. Therefore, there must be 36
houses on the other side of the road before house number 84.
So the first house on the other side of the road is house number 84 36 = 48. Therefore, the
last house on the first side is house number 47.
Each house on one side has a house directly across from it on the other side. Since there are 47
houses on one side, there are 47 houses on the other side and there are a total of 47 2 = 94
houses on Consecutive Crescent.

Solution 2
In this solution, a variable is introduced to help in the argument.
Suppose there are n houses on one side of Consecutive Crescent. Then there are a total of
2 n = 2n houses on the street. House 1 is opposite house n + 1, house 2 is opposite house
n + 2, house 3 is opposite house n + 3, and so on. (The . . . below represent the houses in
between house 3 and house 37 and again the houses between house 37 and house n.) House 37
is opposite house 84 and house n is opposite house 2n.
1
2
3 ...
n + 1 n + 2 n + 3 ...

37 . . . n
84 . . . 2n

There are 36 houses before house 37. Therefore, there must be 36 houses before house 84.
So the first house in the second row is house number 84 36 = 48. This is house n + 1.
Therefore, the last house in the first row, house n, is house number 47.
So there are 47 2 = 94 houses on Consecutive Crescent.

WWW.C E M C .U WAT E R LO O.C A | T h e C E N T R E fo r E D U C AT I O N i n M AT H E M AT I C S a n d CO M P U T I N G

Problem of the Week


Problem C
Eureka!
Eureka literally translated means I have found it!.
Mr. Archimedes would often start his classes with Eureka problems. His
students would be challenged to dig in to solve a problem and when they felt
that they had solved it correctly, they would jump up and say, Eureka!.
During one such class, Mr. Archimedes wrote the following problem on the
board.
I am looking for a positive integer with exactly eight positive divisors. Two of
the divisors are 21 and 35. What is the number?
Solve Mr. Archimedes Eureka problem.

!"#$%&
A divisor of n is an integer that divides exactly into n.
For example:
3 is a divisor of 18 since, 18 3 = 6.
4 is not a divisor of 18, since 18 4 = 4.5.

WWW.C E M C .U WAT E R LO O.C A | T h e C E N T R E fo r E D U C AT I O N i n M AT H E M AT I C S a n d CO M P U T I N G

Problem of the Week


Problem C and Solution
Eureka
Problem
Eureka literally translated means I have found it!. Mr. Archimedes would
often start his classes with Eureka problems. His students would be challenged
to dig in to solve a problem and when they felt that they had solved it
correctly, they would jump up and say, Eureka!. During one such class, Mr.
Archimedes wrote the following problem on the board. I am looking for a
positive integer with exactly eight positive divisors. Two of the divisors are 21
and 35. What is the number? Solve Mr. Archimedes Eureka problem.
Solution
Let n represent the number that is the solution to Mr. Archimedes Eureka
problem.
We know that four of the positive divisors of n are 1, 21, 35 and n. In our
solution we will first find the remaining four positive divisors and then
determine n.
Since 21 is a divisor of n and 21 = 3 7, 3 and 7 must also be divisors of n.
Since 35 is a divisor of n and 35 = 5 7, 5 must also be a divisor of n.
Since 3 is a divisor of n and 5 is a divisor of n, and since 3 and 5 have no
common divisors, 3 5 = 15 must also be a divisor n.
We have found all eight of the positive divisors of Mr. Archimedes number.
The positive divisors are 1, 3, 5, 7, 15, 21, 35 and n. We now need to
determine n. From the list of divisors, we see that the prime factors of n are 3,
5 and 7. Therefore n = 3 5 7 = 105.
Therefore, the number that satisfies the conditions of Mr. Archimedes
problem is 105. Eureka!

WWW.C E M C .U WAT E R LO O.C A | T h e C E N T R E fo r E D U C AT I O N i n M AT H E M AT I C S a n d CO M P U T I N G

Problem of the Week


Problem C
Statistics for the New Year
Integers greater than 1000 are created using the digits 2, 0, 1, 4 exactly once in
each integer.
What is the difference between the average of the largest and smallest of these
integers and the median of all the integers that can be created this way?

WWW.C E M C .U WAT E R LO O.C A | T h e C E N T R E fo r E D U C AT I O N i n M AT H E M AT I C S a n d CO M P U T I N G

Problem of the Week


Problem C and Solution
Statistics for the New Year
Problem
Integers greater than 1000 are created using the digits 2, 0, 1, 4 exactly once in each integer.
What is the difference between the average of the largest and smallest of these integers and the
median of all the integers that can be created this way?

Solution
First, we will determine the average of the largest and smallest numbers.
With a given set of four digits, the largest possible integer that can be formed puts the largest
digit in the thousands place, the second largest digit in the hundreds place, the third largest
digit in the tens place, and the smallest digit in the units place. That is, the largest integer is
formed by putting the digits in decreasing order from the thousands place to the units place.
This is because the largest digit can make the largest contribution in the place with the most
value. Thus, the largest integer that can be formed with the digits 2, 0, 1, 4 is 4210.
Similarly, with a given set of digits the smallest integer is formed by putting the digits in
increasing order from the thousands place to the units place. Here, there is an added wrinkle
that the integer must be at least 1000. Therefore, the thousands digit is at least 1. The
smallest integer of this type that can be made uses a thousands digit of 1, and then lists the
remaining digits in increasing order. This integer is 1024.
Therefore, average of the largest and smallest numbers is

4210+1024
2

5234
2

= 2617.

Now, lets determine the median of all of the numbers that can be created.
Since the thousands digit is at least 1, the smallest numbers will begin with a 1. The
remaining digits will be 0, 2 and 4, in some order. There are six possibilities for the remaining
digits: 024, 042, 204, 240, 402, or 420. Therefore, there are six numbers that begin with a 1.
Similarly, there are six numbers that will begin with a 2 and six numbers that will begin with a
4. Therefore, there are 18 different numbers that can be created this way. The smallest six
numbers will begin with a 1 and the largest six numbers will begin with a 4. The middle six
numbers will begin with a 2 and the median of all of the numbers will equal the median of the
middle six numbers. The middle six numbers, from smallest to largest, are
2014, 2041, 2104, 2140, 2401, 2410
Since there is an even number of numbers, the median of these six numbers is the average of
= 4244
= 2122. Therefore, the median of all of the numbers is
the middle two, which is 2104+2140
2
2
2122. (Alternatively, we could have listed out all 18 numbers that can be created this way in
order to determine the median.)
The difference between the average of the largest and smallest numbers and the median of all
numbers is therefore 2617 2122 = 495.

WWW.C E M C .U WAT E R LO O.C A | T h e C E N T R E fo r E D U C AT I O N i n M AT H E M AT I C S a n d CO M P U T I N G

Problem of the Week


Problem C
Out of Sight
On a certain day, the visibility at sea is 5 km. Boats A and B are 200 m apart
and travelling in opposite directions, each at a different, constant speed. The
two boats are in sight of one another for 15 minutes.
If boat A is travelling at a constant speed of 8 km/h, how fast is boat B
travelling, in km/h?

Boat A

Boat B

WWW.C E M C .U WAT E R LO O.C A | T h e C E N T R E fo r E D U C AT I O N i n M AT H E M AT I C S a n d CO M P U T I N G

Problem of the Week


Problem C and Solution
Out of Sight

Problem
On a certain day, the visibility at sea is 5 km. Boats A and B are 200 m apart
and travelling in opposite directions, each at a different, constant speed. The
two boats are in sight of one another for 15 minutes. If boat A is travelling at
a constant speed of 8 km/h, how fast is boat B travelling, in km/h?
Solution
Boat A is travelling at a constant speed of 8 km/h.
Using the formula distance = speed time, in 15 minutes boat A will travel
km 15

h = 2 km.
8
h
60
The visibility at sea is 5 km, thus boats A and B will be in sight of one
another until they are 5 km apart. We are given that boats A and B are in
sight of one another for 15 minutes. Thus, after 15 minutes boats A and B
must be 5 km apart.
Since boats A and B start out 200 m = 0.2 km apart and boat A travels 2 km
in 15 minutes, boat B must travel 5 0.2 2 = 2.8 km in 15 minutes.
Since boat B travelled 2.8 km in 15 minutes, using the formula
speed = distance time, boat B must have been travelling at a speed of
15
60
2.8 km
h = 2.8
= 11.2 km/h.
60
15
Therefore, boat B is travelling at a speed of 11.2 km/h.

WWW.C E M C .U WAT E R LO O.C A | T h e C E N T R E fo r E D U C AT I O N i n M AT H E M AT I C S a n d CO M P U T I N G

Problem of the Week


Problem C
Framed
Four rectangular strips of wood, each 30 cm long and 3 cm wide, are arranged
to form the outer section of a picture frame, as shown.
Determine the area inside the wooden frame.

WWW.C E M C .U WAT E R LO O.C A | T h e C E N T R E fo r E D U C AT I O N i n M AT H E M AT I C S a n d CO M P U T I N G

Problem of the Week


Problem C and Solutions
Framed
Problem
Four rectangular strips of wood, each 30 cm long and 3 cm wide, are arranged
to form the outer section of a picture frame, as shown. Determine the area
inside the wooden frame.

Solution 1
The inner square has side length 30 3 = 27 cm.
Area = Length Width = 27 27 = 729 cm2 .

30

27

Therefore, the enclosed area is 729 cm2 .


3

Solution 2

Each rectangle has area 30 3 = 90 cm2 .


The outer square has a side length equal to
30 + 3 = 33 cm.

33

30

The area of the outer square is therefore


33 33 = 1089 cm2 .
The area of the inner square is equal to the area
of the outer square minus the areas of the four
rectangles.

30

Therefore, the area of the inner square is 1089 4 90 = 729 cm2 .

33

WWW.C E M C .U WAT E R LO O.C A | T h e C E N T R E fo r E D U C AT I O N i n M AT H E M AT I C S a n d CO M P U T I N G

Problem of the Week


Problem C
Our Fair Share
Four students share a $1000 prize for a project. They decide to split the money
based on the amount of work each team member contributed to the project.
Alison and Bob receive 18 and 15 of the total. Carl receives the average of what
Alison and Bob receive. Dana receives the remainder.
Determine the fraction of the prize money that Dana receives.

WWW.C E M C .U WAT E R LO O.C A | T h e C E N T R E fo r E D U C AT I O N i n M AT H E M AT I C S a n d CO M P U T I N G

Problem of the Week


Problem C and Solutions
Our Fair Share
Problem
Four students share a $1000 prize for a project. They decide to split the money
based on the amount of work each team member contributed to the project.
Alison and Bob receive 18 and 15 of the total. Carl receives the average of what
Alison and Bob receive. Dana receives the remainder. Determine the fraction
of the prize money that Dana receives.
Solution
In the first solution, we solve the problem by working with the fractions and
without calculating the amount of money each person receives. In the second
solution, we determine the fraction of the winnings Dana receives by calculating
the amount of money Dana receives and dividing by the total prize money.
Solution 1
1
8
1
1
8 + 5

Carl receives the average of

and

1
5

of the prize money.

5
40

8
13
+ 40
13
Therefore, Carl receives
=
= 40 =
of the total prize money.
2
2
2
80
Dana receives the remainder of the prize money.
80
10
16
13
41
Therefore, Dana receives 1 81 15 13
80 = 80 80 80 80 = 80 of the total
prize money.

Solution 2
Alison receives 81 of the prize money, so she receives 81 $1000 = $125.
Bob receives 15 of the prize money, so he receives 51 $1000 = $200.
Carl receives the average of what Alison and Bob receive.
Therefore, Carl receives $125+$200
= $325
2
2 = $162.50.
Dana receives the remainder of the prize money.
Therefore, Dana receives $1000 $125 $200 $162.50 = $512.50.
That is, Dana receives

512.50
1000

5125
10000

41
80

of the total prize money.

WWW.C E M C .U WAT E R LO O.C A | T h e C E N T R E fo r E D U C AT I O N i n M AT H E M AT I C S a n d CO M P U T I N G

Problem of the Week


Problem C
Just an Average Box
In a sequence of 6 numbers, every number after the first two is the average of
the previous two numbers.
The 4th number in the sequence is 22 and the 6th number in the sequence is 45.
Determine all 6 numbers in the sequence.

? ? ? 22 ? 45

WWW.C E M C .U WAT E R LO O.C A | T h e C E N T R E fo r E D U C AT I O N i n M AT H E M AT I C S a n d CO M P U T I N G

Problem of the Week


Problem C and Solutions
Just an Average Box
Problem
In a sequence of 6 numbers, every number after the first two is the average of the previous two
numbers. The 4th number in the sequence is 22 and the 6th number in the sequence is 45.
Determine all 6 numbers in the sequence.

? ? ? 22 ? 45
Note
If c is the average of two numbers a and b, then

a+b
= c.
2

It follows that a + b = 2 c.
This idea is used in both solutions.

Solution 1
In the first solution, we solve the problem by working backwards.
Since the 6th number in the sequence is equal to the average of the two previous numbers, the
6th number must be the average of the 4th and 5th numbers.
So the sum of the 4th and 5th numbers must be 2 times the 6th number, or 2 45 = 90.
Therefore, the 5th number is 90 22 = 68.
We now determine the 3rd number. The 5th number in the sequence is the average of the 3rd
and 4th numbers. So the sum of the 3rd and 4th numbers is 2 times the 5th number, or
2 68 = 136. Therefore, the 3rd number is 136 22 = 114.
We now determine the 2nd number. The 4th number in the sequence is the average of the 2nd
and 3rd numbers. So the sum of the 2nd and 3rd numbers is 2 times the 4th number, or
2 22 = 44. Therefore, the 2nd number is 44 114 = 70.
We now determine the 1st number. The 3rd number in the sequence is the average of the 1st
and 2nd numbers. So the sum of the 1st and 2nd numbers is 2 times the 3rd number, or
2 114 = 228. Therefore, the 1st number is 228 (70) = 228 + 70 = 298.
The sequence of 6 numbers is 298, 70, 114, 22, 68, 45.
We can indeed check that in this sequence each number after the first two is equal to the
average of the previous two numbers.

WWW.C E M C .U WAT E R LO O.C A | T h e C E N T R E fo r E D U C AT I O N i n M AT H E M AT I C S a n d CO M P U T I N G

Solution 2
We will now present a similar, but more algebraic solution.
Let the sequence be

22

45

where a represents the 1st number, b represents the 2nd number, c represents the 3rd number
and d represents the 5th number in the sequence.
We again solve this problem by working backwards.
Since the 6th number in the sequence is equal to the average of the 4th and 5th numbers, we
have 45 = 22+d
. Multiplying both sides by 2, we obtain 22 + d = 45 2 = 90. Rearranging,
2
d = 90 22 = 68.
Therefore, the 5th number in the sequence is 68.
We now determine the 3rd number. Since the 5th number in the sequence is equal to the
. Multiplying both sides by 2, we obtain
average of the 3rd and 4th numbers, we have 68 = c+22
2
c + 22 = 68 2 = 136. Rearranging, c = 136 22 = 114.
Therefore, the 3rd number in the sequence is 114.
We now determine the 2nd number. Since the 4th number in the sequence is equal to the
average of the 2nd and 3rd numbers, we have 22 = b+114
. Multiplying both sides by 2, we obtain
2
b + 114 = 22 2 = 44. Rearranging, b = 44 114 = 70.
Therefore, the 2nd number in the sequence is 70.
We now determine the 1st number. Since the 3rd number in the sequence is equal to the
a + (70)
. Multiplying both sides by 2, we
average of the 1st and 2nd numbers, we have 114 =
2
obtain a + (70) = 114 2 = 228. Rearranging, a = 228 + 70 = 298.
Therefore, the 1st number in the sequence is 298.
Therefore, the sequence of 6 numbers is 298, 70, 114, 22, 68, 45.
We can indeed check that in this sequence each number after the first two is equal to the
average of the previous two numbers.

298 -70 114 22

68

45

WWW.C E M C .U WAT E R LO O.C A | T h e C E N T R E fo r E D U C AT I O N i n M AT H E M AT I C S a n d CO M P U T I N G

Problem of the Week


Problem C
Pip Pip
A domino tile is a rectangular tile with a line dividing its face into two square
ends. Each end is marked with a number of dots (also called pips) or is blank.

The domino on the left is a [3, 5] domino, since there are 3 pips on one end and
5 pips on the other end. The domino in the middle is a [0, 4] domino, since
there are 0 pips on one end and 4 pips on the other end. The domino on the
right is a [3, 3] domino, since there are 3 pips on one end and 3 pips on the
other end.
We can also rotate the domino tiles:

The domino on the left is a [5, 3] domino. However, since each tile has just
been rotated, [5, 3] and [3, 5] represent the same domino. Similarly, the domino
in the middle is a [4, 0] domino. Note that [4, 0] and [0, 4] represent the same
domino.
A 2-set of dominoes contains all the tiles with the number of pips on any end
ranging from 0 to 2, and no two dominoes can be the same. A 2-set of
dominoes has the following 6 tiles: [0, 0], [0, 1], [0, 2], [1, 1], [1, 2], [2, 2]. (Notice
that the three dominoes [1, 0], [2, 0] and [2, 1] are not listed because they are
the same as the three dominoes [0, 1], [0, 2] and [1, 2]).
A 10-set of dominoes contains all the tiles with the number of pips on any end
ranging from 0 to 10, and no two dominoes can be the same. How many tiles
are in a 10-set of dominoes?

WWW.C E M C .U WAT E R LO O.C A | T h e C E N T R E fo r E D U C AT I O N i n M AT H E M AT I C S a n d CO M P U T I N G

Problem of the Week


Problem C and Solution
Pip Pip
Problem
A 10-set of dominoes contains all the tiles with the number of pips on any end ranging from 0
to 10, and no two dominoes can be the same. How many tiles are in a 10-set of dominoes?

Solution
Since rotating a domino tile does not change the domino, lets orient each tile so that the
smallest number is always on the left end of the tile.
For each possible number on the left side of the domino, we examine the possible numbers that
can occur on the right side of the domino and compile this information in a table.
Number on Left Side
0
1
2
3
4
5
6
7
8
9
10

Possible Numbers on Right Side


0, 1, 2, 3, 4, 5, 6, 7, 8, 9, 10
1, 2, 3, 4, 5, 6, 7, 8, 9, 10
2, 3, 4, 5, 6, 7, 8, 9, 10
3, 4, 5, 6, 7, 8, 9, 10
4, 5, 6, 7, 8, 9, 10
5, 6, 7, 8, 9, 10
6, 7, 8, 9, 10
7, 8, 9, 10
8, 9, 10
9, 10
10

Total Number of Dominoes


11
10
9
8
7
6
5
4
3
2
1

Therefore, the total number of dominoes in a 10-set is


1 + 2 + 3 + 4 + 5 + 6 + 7 + 8 + 9 + 10 + 11 = 66.
Did you know?
Do you know the quick way to calculate the sum 1 + 2 + 3 + 4 + 5 + 6 + 7 + 8 + 9 + 10 + 11?
(11)(11 + 1)
1 + 2 + 3 + 4 + 5 + 6 + 7 + 8 + 9 + 10 + 11 =
. Why is this true?
2
Let S represent the sum.
S = 1 + 2 + 3 + 4 + 5 + 6 + 7 + 8 + 9 + 10 + 11
S = 11 + 10 + 9 + 8 + 7 + 6 + 5 + 4 + 3 + 2 + 1
2S

12

12

12

12

12

12

12

12

12

(11)(12)
2S = 11(12) and so S =
= 66.
2
In general, it can be shown that if n is a positive integer, then the sum of the integers
(n)(n + 1)
from 1 to n is
. In other words, 1 + 2 + 3 + . . . + n = n(n+1)
.
2
2

12

12

WWW.C E M C .U WAT E R LO O.C A | T h e C E N T R E fo r E D U C AT I O N i n M AT H E M AT I C S a n d CO M P U T I N G

10

2014

Problem of the Week


Problem C
Happy Palindrome Day!

The date, April 10, 2014, is palindromic when the date is written in the form
month-day-year, 4102014. The date, October 4, 2014, is also a palindrome
when the date is written in the form day-month-year, 4102014. A palindrome
is a word, phrase, sentence, or number that reads the same forwards and
backwards. In honour of the first Palindrome Day of 2014 we offer the
following problem:
The number 14 741 is a five-digit palindromic number. Determine the largest
five-digit palindromic number which can be divided exactly by 15.
It may be helpful to note that a number is divisible by 3 if the sum of its digits
is divisible by 3. For example, 15 972 is divisible by 3 since
1 + 5 + 9 + 7 + 2 = 24 and 24 is divisible by 3.

WWW.C E M C .U WAT E R LO O.C A | T h e C E N T R E fo r E D U C AT I O N i n M AT H E M AT I C S a n d CO M P U T I N G

10

2014

Problem of the Week


Problem C and Solution
Happy Palindrome Day!

Problem
The date, April 10, 2014, is palindromic when the date is written in the form
month-day-year, 4102014. The date, October 4, 2014, is also a palindrome
when the date is written in the form day-month-year, 4102014. A palindrome
is a word, phrase, sentence, or number that reads the same forwards and
backwards. In honour of the first Palindrome Day of 2014 we offer the following
problem: The number 14 741 is a five-digit palindromic number. Determine the
largest five-digit palindromic number which can be divided exactly by 15.
Solution
We are looking for a five-digit number of the form abcba.
For a number to be divisible by 15, it must be divisible by both 3 and 5.
To be divisible by 5, a number must end in 0 or 5. If the required number ends
in 0, it must also begin with 0 in order to be a palindrome. But the number
0bcb0 is not a five-digit number. Therefore, the number cannot end in a 0 and
hence must start and end with a 5. The required number looks like 5bcb5.
For a number to be divisible by 3, the sum of its digits must be divisible by 3.
Since we want the largest possible number, let b = 9 in 5bcb5. We must find
the largest value of c so that 59c95 is divisible by 3. The sum of the digits is
5 + 9 + c + 9 + 5 = c + 28 and c can take on any integer value from 0 to 9. It
follows that c + 28 can take on integer values from 28 to 37. The largest
number in this range divisible by 3 is 36 so c + 28 = 36 and c = 8.
The largest five-digit palindromic number exactly divisible by 15 is 59 895.
(As an aside, the smallest five-digit palindromic number exactly divisible by 15
is 50 205. And there are only 33 five-digit palindromic numbers exactly divisible
by 15. The verification of this is left as a possible extension for the solver.)

WWW.C E M C .U WAT E R LO O.C A | T h e C E N T R E fo r E D U C AT I O N i n M AT H E M AT I C S a n d CO M P U T I N G

Problem of the Week


Problem C
Going My Way?
In a computer game, Super Maria travels through the maze shown from Start,
S, to Finish, F . She is allowed to travel only east, south, or southeast along a
path.
How many different routes can Maria take from Start to Finish?

S
N

S
F

WWW.C E M C .U WAT E R LO O.C A | T h e C E N T R E fo r E D U C AT I O N i n M AT H E M AT I C S a n d CO M P U T I N G

Problem of the Week


Problem C and Solution
Going My Way?
Problem
In a computer game, Super Maria travels
through the maze shown from Start, S, to
Finish, F . She is allowed to travel only east,
south, or southeast along a path. How many
different routes can Maria take from Start to
Finish?

S
N

S
F

Solution
We could solve this problem by tracing out
different routes and counting how many we
find. We will set up a systematic approach to
doing so, to ensure that we do not miss any
routes.
We begin by labelling the seven unlabelled
intersections in the maze as A, B, C, D, E, G,
and H, as shown.

Since Maria can only travel east, south, or southeast along a path, starting at
S she has only two choices as to where to go next: A or C.

WWW.C E M C .U WAT E R LO O.C A | T h e C E N T R E fo r E D U C AT I O N i n M AT H E M AT I C S a n d CO M P U T I N G

Case 1: Maria travels from S to A

Again, since Maria can only travel east, south, or


southeast along a path, she has only two choices as to
where to go next: B or D.
If Maria travels to B, then since she can only travel east,
south or southeast, she must go to E next, followed by
F . Therefore, one route from S to F is SABEF .
If Maria travels to D, then since she can travel east,
south or southeast, she can go to E, F or H next.

If she travels from D to E, she must then go to F . Therefore, one route from S to F is
SADEF .
If she travels from D to F , we have found another route. Therefore, one route from S to
F is SADF .
If she travels from D to H, she must then go to F . Therefore, one route from S to F is
SADHF .
Therefore, there are four routes from S to F in which Maria first travels from S to A. They
are SABEF , SADEF , SADF , and SADHF .
Case 2: Maria travels from S to C
Since Maria can travel east, south or southeast, she could travel to D, H or G next.
If she travels from C to D, she again has 3 choices: travel from D to E, F or H. We get
three different routes: SCDEF , SCDF and SCDHF .
If she travels from C to H, from H she must go to F . We have found another route,
SCHF .
If she travels from C to G, she must then go to H then F . We have found another route,
SCGHF .
Therefore, there are five routes from S to F in which Maria first travels from S to C. They are
SCDEF , SCDF , SCDHF , SCHF , and SCGHF .
In total, there are 4 + 5 = 9 different routes that Maria can take from S to F .
For further thought:
How would your answer change if Maria were allowed to also go southwest? (She is never
allowed to travel north, northeast, northwest, or west.)

WWW.C E M C .U WAT E R LO O.C A | T h e C E N T R E fo r E D U C AT I O N i n M AT H E M AT I C S a n d CO M P U T I N G

Problem of the Week


Problem C
Going with the Traffic Flow
In a traffic study done on a busy highway, it was found that 40% of all cars
contained two or more people. Of those cars containing only one person, 25%
contained a male.
Determine the percentage of all cars which contained exactly one female and
no male.

WWW.C E M C .U WAT E R LO O.C A | T h e C E N T R E fo r E D U C AT I O N i n M AT H E M AT I C S a n d CO M P U T I N G

Problem of the Week


Problem C and Solutions
Going with the Traffic Flow
Problem
In a traffic study done on a busy highway, it was found that 40% of all cars
contained two or more people. Of those cars containing only one person, 25%
contained a male. Determine the percentage of all cars which contained exactly
one female and no male.
Solution 1
Lets suppose there were 100 cars on the highway.
If 40% of the cars contained two or more people, that means that
0.4 100 = 40 cars contained two or more people. Therefore, 100 40 = 60
cars contained exactly one person (a car cannot be moving on a highway if
there is less than 1 person in it!).
Of the cars containing only one person, 25% contained a male. Therefore,
0.25 60 = 15 cars with only one person contained a male and 60 15 = 45
contained a female.
Therefore, the percentage of all cars which contained exactly one female and no
45
male is
100% = 45%.
100
Solution 2
If 40% of the cars contained two or more people, then 60% of the cars
contained one person.
Of the cars containing only one person, 25% contained a male. Therefore, of
the cars containing only one person, 75% contained a female.
Since 0.75 0.60 = 0.45, 45% of all cars contained exactly one female.

WWW.C E M C .U WAT E R LO O.C A | T h e C E N T R E fo r E D U C AT I O N i n M AT H E M AT I C S a n d CO M P U T I N G

Problem of the Week


Problem C
How Does Your Garden Grow?
Mr. Digme planted five rose plants in a row along one side of his property.
He then planted one tulip plant in each of the spaces in the row between the
roses.
Next, he planted one daffodil plant in each of the spaces between the plants
already in the row.
He then repeated this procedure with petunias, then marigolds, and finally
with lilies.
Determine the total number of plants in the row.

WWW.C E M C .U WAT E R LO O.C A | T h e C E N T R E fo r E D U C AT I O N i n M AT H E M AT I C S a n d CO M P U T I N G

Problem of the Week


Problem C and Solution
How Does Your Garden Grow?
Problem
Mr. Digme planted five rose plants in a row along one side of his property. He
then planted one tulip plant in each of the spaces in the row between the roses.
Next, he planted one daffodil plant in each of the spaces between the plants
already in the row. He then repeated this procedure with petunias, then
marigolds, and finally with lilies. Determine the total number of plants in the
row.
Solution
After planting 5 roses, there were four spaces between the plants. So, Mr.
Digme then plants 4 tulips. At this point he has planted 5 + 4 = 9 plants.
Since there are now 9 plants, there are 8 spaces between plants. So, Mr. Digme
plants 8 daffodil plants. At this point he has planted 9 + 8 = 17 plants.
Since there are now 17 plants, there are 16 spaces between plants. So, Mr.
Digme plants 16 petunia plants. At this point he has planted 17 + 16 = 33
plants.
Since there are now 33 plants, there are 32 spaces between plants. So, Mr.
Digme plants 32 marigold plants. At this point he has planted 33 + 32 = 65
plants.
Finally, since there are now 65 plants, there are 64 spaces between plants. So,
Mr. Digme plants 64 lily plants. At this point he has planted 65 + 64 = 129
plants.
Therefore, Mr. Digme planted a total of 129 plants in the row.
Extension:
You may have noticed a pattern in the total number of plants after each new
plant is introduced. If Mr. Digme plants flowers in this way using n different
types of flowers, there will be a total of 2n+1 + 1 plants in the row.
Can you see why? In Mr. Digmes garden there were 6 different types of
plants. Verify that the formula is correct when n = 6.

WWW.C E M C .U WAT E R LO O.C A | T h e C E N T R E fo r E D U C AT I O N i n M AT H E M AT I C S a n d CO M P U T I N G

Problem of the Week


Problem C
Looking for Nothing
The product of the positive integers 1 to 6 is
6 5 4 3 2 1 = 720
and can be written in an abbreviated form as 6!. We say, 6 factorial . So
6! = 720.
The product of the positive integers from 1 to 12 is
12 11 10 . . . 3 2 1 = 479 001 600
and can be written in an abbreviated form as 12!. We say, 12 factorial .
The represents the product of all of the missing integers between 10 and 3.
For a positive integer n, the product of the positive integers from 1 to n is n!.
Find the smallest possible value of n such that n! ends in exactly six zeroes.

WWW.C E M C .U WAT E R LO O.C A | T h e C E N T R E fo r E D U C AT I O N i n M AT H E M AT I C S a n d CO M P U T I N G

Problem of the Week


Problem C and Solution
Looking for Nothing
Problem
The product of the positive integers 1 to 6 is 6 5 4 3 2 1 = 720. and
can be written in an abbreviated form as 6!. We say, 6 factorial . So 6! = 720.
The product of the positive integers from 1 to 12 is
12 11 10 . . . 3 2 1 = 479 001 600
and can be written in an abbreviated form as 12!. We say, 12 factorial .
The represents the product of all of the missing integers between 10 and 3.
For a positive integer n, the product of the positive integers from 1 to n is n!.
Find the smallest possible value of n such that n! ends in exactly six zeroes.
Solution
We start by examining the first few factorials:
1! = 1
2! = 2 1 = 2
3! = 3 2 1 = 6
4! = 4 3 2 1 = 24
5! = 5 4 3 2 1 = 120
6! = 6 (5 4 3 2 1) = 6 5! = 6(120) = 720
7! = 7 (6 5 4 3 2 1) = 7 6! = 7(720) = 5040
8! = 8 (7 6 5 4 3 2 1) = 8 7! = 8(5040) = 40 320
9! = 9 (8 7 6 5 4 3 2 1) = 9 8! = 9(40 320) = 362 880
10! = 10 (9 8 7 6 5 4 3 2 1) = 10 9! = 10(362 880) = 3 628 800
These numbers are getting very large and soon will not fit on the display of a
standard calculator. So, lets look at what is going on.

WWW.C E M C .U WAT E R LO O.C A | T h e C E N T R E fo r E D U C AT I O N i n M AT H E M AT I C S a n d CO M P U T I N G

We observe that 5! ends in 0 and 10! ends in 00. Notice that the number of
zeros at the end of the number increased by one at each of 5! and at 10!. Why
is this?
A zero is added to the end of a number when we multiply by 10. Multiplying a
number by 10 is the same as multiplying a number by 2 and then by 5, or by 5
and then by 2, since 2 5 = 10 and 5 2 = 10. We must determine the next
time we multiply by 2 and 5 (in some order), to know the next time the
number of zeros at the end of the number increases again. Every time we
multiply by an even number we are multiplying by at least one more 2. There
are less multiples of five. Each multiple of five will affect the number of zeroes
at the end of the product.
Multiplying by 11, 12, 13, and 14 increases the number of 2s we multiply by
but not the number of 5s. So the number of zeros at the end of the product
does not change. The next time we multiply by a 5 is when we multiply by 15
since 15 = 5 3. So 15! will end in exactly three zeroes, 000.
Multiplying by 16, 17, 18, and 19 increases the number of 2s we multiply by
but not the number of 5s. So the number of zeros at the end of the product
does not change. The next time we multiply by a 5 is when we multiply by 20
since 20 = 4 5. So 20! will end in exactly four zeroes, 0000.
Multiplying by 21, 22, 23, and 24 increases the number of 2s we multiply by
but not the number of 5s. The next time we multiply by a 5 is when we
multiply by 25. In fact, multiplying by 25 is the same as multiplying by 5 twice
since 25 = 5 5. So when we multiply by 25, we will increase the number of
zeros on the end of the product by two. So 25! will end in exactly six zeroes,
000 000.
Then, for n! to end in six zeroes, the smallest value of n is 25. That is, 25! is
the smallest factorial that ends in exactly six zeroes. (It could be noted that
26!, 27!, 28!, and 29! also end in six zeroes.)

WWW.C E M C .U WAT E R LO O.C A | T h e C E N T R E fo r E D U C AT I O N i n M AT H E M AT I C S a n d CO M P U T I N G

Problem of the Week


Problem C
Neighbours
The houses on Consecutive Crescent are arranged so that each house is directly
opposite another house.
The houses are numbered consecutively 1, 2, 3, . . . and so on along one side.
Once the end of that side of the crescent is reached, the consecutive numbering
continues at the house on the other side of the street opposite house number 1.
The consecutive numbering continues along this second side until the last
house is numbered.
If there were ten houses on the crescent, they would be numbered as shown in
the diagram below.
However, on the actual crescent, when the residents of house number 37 look
directly across the street they see house number 84. How many houses are on
Consecutive Crescent?

CONSECUTIVE CRESCENT

10

WWW.C E M C .U WAT E R LO O.C A | T h e C E N T R E fo r E D U C AT I O N i n M AT H E M AT I C S a n d CO M P U T I N G

Problem of the Week


Problem C and Solutions
Neighbours
Problem
The houses on Consecutive Crescent are arranged so that each house is directly opposite
another house. The houses are numbered consecutively 1, 2, 3, . . . and so on along one side.
Once the end of that side of the crescent is reached, the consecutive numbering continues at
the house on the other side of the street opposite house number 1. The consecutive numbering
continues along this second side until the last house is numbered. When the residents of house
number 37 look directly across the street they see house number 84. How many houses are on
Consecutive Crescent?

Solution 1
In this solution we will use logic to reason the number of houses on Consecutive Crescent.
There are 36 houses before house 37 on the one side of the road. Therefore, there must be 36
houses on the other side of the road before house number 84.
So the first house on the other side of the road is house number 84 36 = 48. Therefore, the
last house on the first side is house number 47.
Each house on one side has a house directly across from it on the other side. Since there are 47
houses on one side, there are 47 houses on the other side and there are a total of 47 2 = 94
houses on Consecutive Crescent.

Solution 2
In this solution, a variable is introduced to help in the argument.
Suppose there are n houses on one side of Consecutive Crescent. Then there are a total of
2 n = 2n houses on the street. House 1 is opposite house n + 1, house 2 is opposite house
n + 2, house 3 is opposite house n + 3, and so on. (The . . . below represent the houses in
between house 3 and house 37 and again the houses between house 37 and house n.) House 37
is opposite house 84 and house n is opposite house 2n.
1
2
3 ...
n + 1 n + 2 n + 3 ...

37 . . . n
84 . . . 2n

There are 36 houses before house 37. Therefore, there must be 36 houses before house 84.
So the first house in the second row is house number 84 36 = 48. This is house n + 1.
Therefore, the last house in the first row, house n, is house number 47.
So there are 47 2 = 94 houses on Consecutive Crescent.

WWW.C E M C .U WAT E R LO O.C A | T h e C E N T R E fo r E D U C AT I O N i n M AT H E M AT I C S a n d CO M P U T I N G

Problem of the Week


Problem C
Run for Fun and a Good Cause
For an upcoming charity 5K fun run, Joe, Ann and Henry collected donations.
Joe collected $375 more than his cousin Ann who raised $250 more than her
friend Henry. Together, Joe, Ann and Henry received $2000.
Determine the amount of money that each person raised and the ratio of the
amount of money Henry raised to the amount Ann raised to the amount Joe
raised.

WWW.C E M C .U WAT E R LO O.C A | T h e C E N T R E fo r E D U C AT I O N i n M AT H E M AT I C S a n d CO M P U T I N G

Problem of the Week


Problem C and Solutions
Run for Fun and a Good Cause
Problem
For an upcoming charity 5K fun run, Joe, Ann and Henry collected donations. Joe collected
$375 more than his cousin Ann who raised $250 more than her friend Henry. Together, Joe,
Ann and Henry received $2000. Determine the amount of money that each person raised and
the ratio of the amount of money Henry raised to the amount Ann raised to the amount Joe
raised.

Solution 1
Let $h be the amount of money that Henry collected.
It follows that Ann collected $(h + 250) and Joe collected $(h + 250 + 375).
Then
h + (h + 250) + (h + 250 + 375) = 2000
h + (h + 250) + (h + 625) = 2000
3h + 875 = 2000
3h = 1125
h = 375
So, h + 250 = 625 and h + 250 + 375 = 1000.
Therefore, Henry raised $375, Ann raised $625 and Joe raised $1000.
The ratio of money raised by Henry to the amount raised by Ann to the amount raised by Joe
is 375 : 625 : 1000 = 15 : 25 : 40 = 3 : 5 : 8.

Solution 2
If Joe had raised $625 less and Ann had raised $250 less, each would have raised the same
amount as Henry. Each would then have raised 31 of $(2000 625 250) = 13 of $1 125 = $375.
So Henry raised $375.
Therefore, Ann raised $(375 + 250) = $625 and Joe raised $(625 + 375) = $1000.
The ratio of money raised by Henry to the amount raised by Ann to the amount raised by Joe
is 375 : 625 : 1000 = 15 : 25 : 40 = 3 : 5 : 8.

WWW.C E M C .U WAT E R LO O.C A | T h e C E N T R E fo r E D U C AT I O N i n M AT H E M AT I C S a n d CO M P U T I N G

Problem of the Week


Problem C
And On and On It Goes
1
is the repeating decimal 0.142857.
7
What digit occurs in the 2014th place after the decimal point?

The decimal expansion of

..
.

..
.

WWW.C E M C .U WAT E R LO O.C A | T h e C E N T R E fo r E D U C AT I O N i n M AT H E M AT I C S a n d CO M P U T I N G

Problem of the Week


Problem C and Solution
And On and On It Goes
Problem
1
is the repeating decimal 0.142857.
7
What digit occurs in the 2014th place after the decimal point?

The decimal expansion of

Solution
The digits after the decimal point occur in repeating blocks of the 6 digits,
namely 142857.
2014
2
= 335 , then the 2014th digit after the decimal point occurs after
6
3
335 blocks of the repeating digits have been used.
Since

In 335 blocks of 6 digits, there are 335 6 = 2010 digits in total.


The 2014th digit is 4 digits into the 336th block. Therefore, the 2014th digit
after the decimal point must be 8.

WWW.C E M C .U WAT E R LO O.C A | T h e C E N T R E fo r E D U C AT I O N i n M AT H E M AT I C S a n d CO M P U T I N G

Problem of the Week


Problem C
Just an Average Box
In a sequence of 6 numbers, every number after the first two is the average of
the previous two numbers.
The 4th number in the sequence is 22 and the 6th number in the sequence is 45.
Determine all 6 numbers in the sequence.

? ? ? 22 ? 45

WWW.C E M C .U WAT E R LO O.C A | T h e C E N T R E fo r E D U C AT I O N i n M AT H E M AT I C S a n d CO M P U T I N G

Problem of the Week


Problem C and Solutions
Just an Average Box
Problem
In a sequence of 6 numbers, every number after the first two is the average of the previous two
numbers. The 4th number in the sequence is 22 and the 6th number in the sequence is 45.
Determine all 6 numbers in the sequence.

? ? ? 22 ? 45
Note
If c is the average of two numbers a and b, then

a+b
= c.
2

It follows that a + b = 2 c.
This idea is used in both solutions.

Solution 1
In the first solution, we solve the problem by working backwards.
Since the 6th number in the sequence is equal to the average of the two previous numbers, the
6th number must be the average of the 4th and 5th numbers.
So the sum of the 4th and 5th numbers must be 2 times the 6th number, or 2 45 = 90.
Therefore, the 5th number is 90 22 = 68.
We now determine the 3rd number. The 5th number in the sequence is the average of the 3rd
and 4th numbers. So the sum of the 3rd and 4th numbers is 2 times the 5th number, or
2 68 = 136. Therefore, the 3rd number is 136 22 = 114.
We now determine the 2nd number. The 4th number in the sequence is the average of the 2nd
and 3rd numbers. So the sum of the 2nd and 3rd numbers is 2 times the 4th number, or
2 22 = 44. Therefore, the 2nd number is 44 114 = 70.
We now determine the 1st number. The 3rd number in the sequence is the average of the 1st
and 2nd numbers. So the sum of the 1st and 2nd numbers is 2 times the 3rd number, or
2 114 = 228. Therefore, the 1st number is 228 (70) = 228 + 70 = 298.
The sequence of 6 numbers is 298, 70, 114, 22, 68, 45.
We can indeed check that in this sequence each number after the first two is equal to the
average of the previous two numbers.

WWW.C E M C .U WAT E R LO O.C A | T h e C E N T R E fo r E D U C AT I O N i n M AT H E M AT I C S a n d CO M P U T I N G

Solution 2
We will now present a similar, but more algebraic solution.
Let the sequence be

22

45

where a represents the 1st number, b represents the 2nd number, c represents the 3rd number
and d represents the 5th number in the sequence.
We again solve this problem by working backwards.
Since the 6th number in the sequence is equal to the average of the 4th and 5th numbers, we
have 45 = 22+d
. Multiplying both sides by 2, we obtain 22 + d = 45 2 = 90. Rearranging,
2
d = 90 22 = 68.
Therefore, the 5th number in the sequence is 68.
We now determine the 3rd number. Since the 5th number in the sequence is equal to the
. Multiplying both sides by 2, we obtain
average of the 3rd and 4th numbers, we have 68 = c+22
2
c + 22 = 68 2 = 136. Rearranging, c = 136 22 = 114.
Therefore, the 3rd number in the sequence is 114.
We now determine the 2nd number. Since the 4th number in the sequence is equal to the
average of the 2nd and 3rd numbers, we have 22 = b+114
. Multiplying both sides by 2, we obtain
2
b + 114 = 22 2 = 44. Rearranging, b = 44 114 = 70.
Therefore, the 2nd number in the sequence is 70.
We now determine the 1st number. Since the 3rd number in the sequence is equal to the
a + (70)
. Multiplying both sides by 2, we
average of the 1st and 2nd numbers, we have 114 =
2
obtain a + (70) = 114 2 = 228. Rearranging, a = 228 + 70 = 298.
Therefore, the 1st number in the sequence is 298.
Therefore, the sequence of 6 numbers is 298, 70, 114, 22, 68, 45.
We can indeed check that in this sequence each number after the first two is equal to the
average of the previous two numbers.

298 -70 114 22

68

45

WWW.C E M C .U WAT E R LO O.C A | T h e C E N T R E fo r E D U C AT I O N i n M AT H E M AT I C S a n d CO M P U T I N G

Problem of the Week


Problem C
Pip Pip
A domino tile is a rectangular tile with a line dividing its face into two square
ends. Each end is marked with a number of dots (also called pips) or is blank.

The domino on the left is a [3, 5] domino, since there are 3 pips on one end and
5 pips on the other end. The domino in the middle is a [0, 4] domino, since
there are 0 pips on one end and 4 pips on the other end. The domino on the
right is a [3, 3] domino, since there are 3 pips on one end and 3 pips on the
other end.
We can also rotate the domino tiles:

The domino on the left is a [5, 3] domino. However, since each tile has just
been rotated, [5, 3] and [3, 5] represent the same domino. Similarly, the domino
in the middle is a [4, 0] domino. Note that [4, 0] and [0, 4] represent the same
domino.
A 2-set of dominoes contains all the tiles with the number of pips on any end
ranging from 0 to 2, and no two dominoes can be the same. A 2-set of
dominoes has the following 6 tiles: [0, 0], [0, 1], [0, 2], [1, 1], [1, 2], [2, 2]. (Notice
that the three dominoes [1, 0], [2, 0] and [2, 1] are not listed because they are
the same as the three dominoes [0, 1], [0, 2] and [1, 2]).
A 10-set of dominoes contains all the tiles with the number of pips on any end
ranging from 0 to 10, and no two dominoes can be the same. How many tiles
are in a 10-set of dominoes?

WWW.C E M C .U WAT E R LO O.C A | T h e C E N T R E fo r E D U C AT I O N i n M AT H E M AT I C S a n d CO M P U T I N G

Problem of the Week


Problem C and Solution
Pip Pip
Problem
A 10-set of dominoes contains all the tiles with the number of pips on any end ranging from 0
to 10, and no two dominoes can be the same. How many tiles are in a 10-set of dominoes?

Solution
Since rotating a domino tile does not change the domino, lets orient each tile so that the
smallest number is always on the left end of the tile.
For each possible number on the left side of the domino, we examine the possible numbers that
can occur on the right side of the domino and compile this information in a table.
Number on Left Side
0
1
2
3
4
5
6
7
8
9
10

Possible Numbers on Right Side


0, 1, 2, 3, 4, 5, 6, 7, 8, 9, 10
1, 2, 3, 4, 5, 6, 7, 8, 9, 10
2, 3, 4, 5, 6, 7, 8, 9, 10
3, 4, 5, 6, 7, 8, 9, 10
4, 5, 6, 7, 8, 9, 10
5, 6, 7, 8, 9, 10
6, 7, 8, 9, 10
7, 8, 9, 10
8, 9, 10
9, 10
10

Total Number of Dominoes


11
10
9
8
7
6
5
4
3
2
1

Therefore, the total number of dominoes in a 10-set is


1 + 2 + 3 + 4 + 5 + 6 + 7 + 8 + 9 + 10 + 11 = 66.
Did you know?
Do you know the quick way to calculate the sum 1 + 2 + 3 + 4 + 5 + 6 + 7 + 8 + 9 + 10 + 11?
(11)(11 + 1)
1 + 2 + 3 + 4 + 5 + 6 + 7 + 8 + 9 + 10 + 11 =
. Why is this true?
2
Let S represent the sum.
S = 1 + 2 + 3 + 4 + 5 + 6 + 7 + 8 + 9 + 10 + 11
S = 11 + 10 + 9 + 8 + 7 + 6 + 5 + 4 + 3 + 2 + 1
2S

12

12

12

12

12

12

12

12

12

(11)(12)
2S = 11(12) and so S =
= 66.
2
In general, it can be shown that if n is a positive integer, then the sum of the integers
(n)(n + 1)
from 1 to n is
. In other words, 1 + 2 + 3 + . . . + n = n(n+1)
.
2
2

12

12

WWW.C E M C .U WAT E R LO O.C A | T h e C E N T R E fo r E D U C AT I O N i n M AT H E M AT I C S a n d CO M P U T I N G

10

2014

Problem of the Week


Problem C
Happy Palindrome Day!

The date, April 10, 2014, is palindromic when the date is written in the form
month-day-year, 4102014. The date, October 4, 2014, is also a palindrome
when the date is written in the form day-month-year, 4102014. A palindrome
is a word, phrase, sentence, or number that reads the same forwards and
backwards. In honour of the first Palindrome Day of 2014 we offer the
following problem:
The number 14 741 is a five-digit palindromic number. Determine the largest
five-digit palindromic number which can be divided exactly by 15.
It may be helpful to note that a number is divisible by 3 if the sum of its digits
is divisible by 3. For example, 15 972 is divisible by 3 since
1 + 5 + 9 + 7 + 2 = 24 and 24 is divisible by 3.

WWW.C E M C .U WAT E R LO O.C A | T h e C E N T R E fo r E D U C AT I O N i n M AT H E M AT I C S a n d CO M P U T I N G

10

2014

Problem of the Week


Problem C and Solution
Happy Palindrome Day!

Problem
The date, April 10, 2014, is palindromic when the date is written in the form
month-day-year, 4102014. The date, October 4, 2014, is also a palindrome
when the date is written in the form day-month-year, 4102014. A palindrome
is a word, phrase, sentence, or number that reads the same forwards and
backwards. In honour of the first Palindrome Day of 2014 we offer the following
problem: The number 14 741 is a five-digit palindromic number. Determine the
largest five-digit palindromic number which can be divided exactly by 15.
Solution
We are looking for a five-digit number of the form abcba.
For a number to be divisible by 15, it must be divisible by both 3 and 5.
To be divisible by 5, a number must end in 0 or 5. If the required number ends
in 0, it must also begin with 0 in order to be a palindrome. But the number
0bcb0 is not a five-digit number. Therefore, the number cannot end in a 0 and
hence must start and end with a 5. The required number looks like 5bcb5.
For a number to be divisible by 3, the sum of its digits must be divisible by 3.
Since we want the largest possible number, let b = 9 in 5bcb5. We must find
the largest value of c so that 59c95 is divisible by 3. The sum of the digits is
5 + 9 + c + 9 + 5 = c + 28 and c can take on any integer value from 0 to 9. It
follows that c + 28 can take on integer values from 28 to 37. The largest
number in this range divisible by 3 is 36 so c + 28 = 36 and c = 8.
The largest five-digit palindromic number exactly divisible by 15 is 59 895.
(As an aside, the smallest five-digit palindromic number exactly divisible by 15
is 50 205. And there are only 33 five-digit palindromic numbers exactly divisible
by 15. The verification of this is left as a possible extension for the solver.)

WWW.C E M C .U WAT E R LO O.C A | T h e C E N T R E fo r E D U C AT I O N i n M AT H E M AT I C S a n d CO M P U T I N G

Problem of the Week


Problem C
How Does Your Garden Grow?
Mr. Digme planted five rose plants in a row along one side of his property.
He then planted one tulip plant in each of the spaces in the row between the
roses.
Next, he planted one daffodil plant in each of the spaces between the plants
already in the row.
He then repeated this procedure with petunias, then marigolds, and finally
with lilies.
Determine the total number of plants in the row.

WWW.C E M C .U WAT E R LO O.C A | T h e C E N T R E fo r E D U C AT I O N i n M AT H E M AT I C S a n d CO M P U T I N G

Problem of the Week


Problem C and Solution
How Does Your Garden Grow?
Problem
Mr. Digme planted five rose plants in a row along one side of his property. He
then planted one tulip plant in each of the spaces in the row between the roses.
Next, he planted one daffodil plant in each of the spaces between the plants
already in the row. He then repeated this procedure with petunias, then
marigolds, and finally with lilies. Determine the total number of plants in the
row.
Solution
After planting 5 roses, there were four spaces between the plants. So, Mr.
Digme then plants 4 tulips. At this point he has planted 5 + 4 = 9 plants.
Since there are now 9 plants, there are 8 spaces between plants. So, Mr. Digme
plants 8 daffodil plants. At this point he has planted 9 + 8 = 17 plants.
Since there are now 17 plants, there are 16 spaces between plants. So, Mr.
Digme plants 16 petunia plants. At this point he has planted 17 + 16 = 33
plants.
Since there are now 33 plants, there are 32 spaces between plants. So, Mr.
Digme plants 32 marigold plants. At this point he has planted 33 + 32 = 65
plants.
Finally, since there are now 65 plants, there are 64 spaces between plants. So,
Mr. Digme plants 64 lily plants. At this point he has planted 65 + 64 = 129
plants.
Therefore, Mr. Digme planted a total of 129 plants in the row.
Extension:
You may have noticed a pattern in the total number of plants after each new
plant is introduced. If Mr. Digme plants flowers in this way using n different
types of flowers, there will be a total of 2n+1 + 1 plants in the row.
Can you see why? In Mr. Digmes garden there were 6 different types of
plants. Verify that the formula is correct when n = 6.

Vous aimerez peut-être aussi